You are on page 1of 96
‘A, KINEMATICS OF FLOW > 5.1 INTRODUCTION Kinematics is defined as that branch of science which deals with motion of particles without ‘considering the forces causing the motion. The velocity at any point in a flow field at any time is studied in this branch of fluid mechanics. Once the velocity is known, then the pressure distribution and hence forces acting on the fluid can be determined. In this chapter, the methods of determining. velocity and acceleration are discussed. > 5.2. METHODS OF DESCRIBING FLUID MOTION ‘The fluid motion is described by two methods. They are —(#) Lagrangian Method, and (ii) Eulerian Method. In the Lagrangian method, a single fluid particle is followed during its motion and its velocity, acceleration, density, etc., are described. In case of Eulerian method, the velocity, accelera- tion, pressure, density etc., are described at a point in flow field. The Eulerian method is commonly used in fluid mechanics. > 5.3 TYPES OF FLUID FLOW ‘The fluid flow is classified as : (@) Steady and unsteady flows ; (i) Uniform and non-uniform flows ; iii) Laminar and turbulent flows ; (iv) Compressible and incompressible flows ; (») Rotational and irrotational flows ; and (vi) One, two and three-dimensional flows. 5.3.1 Steady and Unsteady Flows. Stcady flow is defined as that type of flow in which the fluid characteristics like velocity, pressure, density, etc., at a point do not change with time. Thus for steady flow, mathematically, we have 163, 164 Fluid Mechanics av =0,(%) = () -0(%)_ 7° whore (32 ted pln fi Hl Unsteady flow is that type of flow, in which the velocity, pressure or density at a point changes with respect to time. Thus, mathematically, for unsteady flow v (Fo (2) soe 5.3.2. Uniform and Non-uniform Flows. Uniform flow is defined as that type of flow in ‘which the velocity at any given time does not change with respect to space (i.e., length of direction of the flow). Mathematically, for uniform flow S.-° where V= Change of velocity {as = Length of flow in the direction 5. Non-uniform flow is that type of flow in which the velocity at any given time changes with respect to space. Thus, mathematically, for non-uniform flow Sw 5.3.3. Laminar and Turbulent Flows. Laminar flow is defined as that type of flow in which the fluid particles move along well-defined paths or stream line and all the stream-lines are straight and parallel. Thus the particles move in laminas or layers gliding smoothly over the adjacent layer. This type of flow is also called stream-line flow or viscous flow. Turbulent flow is that type of flow in which the fluid particles move in a zig-cag way. Due to the ‘movement of fluid particles in a zig-zag way, the eddies formation takes place which are responsible for high energy loss. For a pipe flow, the type of flow is determined by a non-dimensional number called the Reynold number, where D = Diameter of pipe V = Mean velocity of flow in pipe and v= Kinematic viscosity of fluid. If the Reynold number is less than 2000, the flow is called laminar. If the Reynold number is more than 4000, it is called turbulent flow. If the Reynold number lies between 2000 and 4000, the flow may be laminar or turbulent. 5.3.4 Compressible and Incompressible Flows. Compressible flow is that type of flow in which the density of the fluid changes from point to point or in other words the density (p) is not ‘constant for the fluid. Thus, mathematically, for compressible flow p # Constant Incompressible flow is that type of flow in which the density is constant for the fluid flow. Liquids are generally incompressible while gases are compressible. Mathematically, for incompressible flow p= Constant. [ Kinematics of Flow and Ideal Flow 165 5.3.5 Rotational and Irrotational Flows. Rotational flow is that type of flow in which the fluid particles while flowing along stream-lines, also rotate about their own axis. And if the fluid particles while flowing along stream-lines, do not rotate about their own axis then that type of flow is called irrotational flow. 5.3.6 One-, Two- and Three-Dimensional Flows. One-dimensional flow is that type of flow in which the flow parameter such as velocity isa function of time and one space co-ordinate only, say x. For a steady one-dimensional flow, the velocity isa function of one-space-co-ordinate only. The variation of velocities in other two mutually perpendicular directions is assumed negligible, Hence ‘mathematically, for one-dimensional flow w= fs), v= and w=0 where u, v and w are velocity components in x, y and z directions respectively. ‘Two-dimensional flow is that type of flow in which the velocity is a function of time and two rectangular space co-ordinates say x and y. For a steady two-dimensional flow the velocity isa function of two space co-ordinates only. The variation of velocity in the third direction is negligible. Thus, mathematically for two-dimensional flow N= fC), ¥=fylasy) and w= 0. ‘Three-dimensional flow is that type of flow in which the velocity is a function of time and three ‘mutually perpendicular directions. But for a steady three-dimensional flow the fluid parameters are functions of three space co-ordinates (x, y and z) only. Thus, mathematically, for three-dimensional flow 0 YZ ¥ ffx. Yo 2) and w= fyl% Ys 2) > 5.4 RATE OF FLOW OR DISCHARGE (Q) Itis defined as the quantity of a fluid flowing per second through a section ofa pipe or a channel. For an incompressible fluid (or liquid) the rate of flow or discharge is expressed as the volume of fluid flowing across the section per second. For compressible fluids, the rate of flow is usually expressed as the weight of fluid flowing across the section. Thus () For liquids the units of Q are m’/s or litres/s (ii) For gases the units of Q is kgfis or Newton/s Consider aliquid flowing through a pipe in which A= Cross-seetional area of pipe V = Average velocity of fluid across the section Then discharge Q= AX. 6.1) > 5.5 CONTINUITY EQUATION ‘The equation based on the principle of conservation of mass is called continuity equation. Thus for a fluid flowing through the pipe at all the cross-section, the quantity of fluid per second is constant Consider two cross-sections of a pipe as shown in Fig. 5.1 Let_V; = Average velocity at cross-section 1-1 p, = Density at section 1-1 ‘Ay = Arca of pipe at section 1-1 166 Fluid Mechanics and V2, Py, Ay are corresponding values at section, 2-2, ‘Then rate of flow at section 1-1 = pyA\V, a i Rate of flow at section 2-2 = p,A,V> DIRECTION ‘According 10 law of conservation of mass oF FLOW Rate of flow at section 1-1 = Rate of flow at section 2-2 or PLAY: = PoAVe (52) Equation (5.2) is applicable to the compressible as well as incom- Fig. 51 Fluid flowing through pressible luids and is called Continuity Equation. Ifthe fluid is in- @ pipe. compressible, then p, = p and continuity equation (5.2) reduces to AW, = AV 63) Problem 5.1. The diameters ofa pipe atthe sections I and 2 are 10.cm and 5 em respectively. Find the discharge through the pipe if the velocity of water flowing through the pipe at section 1 is 5 m/s. Determine also the velocity at section 2. Solution. Given : Atsection 1, D,=10em=0.1 m Rope ay Aya DP) =F mis. At section 2, Sem = 0.15 m Ys Smee (15)? = 0.01767 m? Fig. 5.2 (® Discharge through pipe is given by equation (5.1) or O=A,xV, = 0.007854 x 5 = 0.03927 m’ss. Ans. Using equation (5.3), we have AV, = 4,V; AY, _ 0.007854 ‘A, 0.001767 Problem 5.2 A 30 cm diameter pipe, conveying water, branches into two pipes of diameters 20 cm and 15 cm respectively. If the average velocity in the 30 cm diameter pipe is 2.5 mls, find the discharge in this pipe. Also determine the velocity in 15 cm pipe if the average velocity in 20 cm diameter pipe is 2 m/s. Solution, Given : (i) V X 5.0 = 2.22 m/s. Ans. Oe 17 ore aatamieg y= 30cm ® a i Fig. 5.3 Kinematics of Flow and Ideal Flow 167 D, = 30m = 0.30 m Ex se im? 5 x 3= 0.07068 5m = 0.15 m (sy =F x 0.225 = 0.01767 m? Find (i) Discharge in pipe 1 or Q, (i, Velocity in pipe of dia, 15 em or Vs Let Qj, Qs and Q, are discharges in pipe 1, 2 and 3 respectively Then according to continuity equation = 02+ Os a (i The discharge Q, in pipe 1 is given by Q, = AiV; = 0.07068 x 2.5 mvs = 0.1767 m'ss. Ans. (i Value of V5 = AV) = 0.0314 x 2.0 = 0.0628 m/s Substituting the values of Q, and Q, in equation (1) 0.1767 = 0.0628 + 0, z Q = 0.1767 ~ 0.0628 = 0.1139 m*/s But Qs = Ay X Vy = 0.01767 x Vy or 0.1139 = 0.01767 x V5 0.1139 : 6.44 mis. Ans. 0.01767 Problem 5.3 Water flows through a pipe AB 1.2 m diameter at 3 m/s and then passes through a pipe BC 1.5 m diameter. At C, the pipe branches. Branch CD is 0.8 m in diameter and carries one- third of the flow in AB. The flow velocity in branch CE is 2.5 m/s. Find the volume rate of flow in AB, the velocity in BC, the velocity in CD and the diameter of CE. Solution. Given Diameter of pipe AB, Du Velocity of flow through AB, Vi Dia. of pipe BC, Dac Dia. of branched pipe CD, Dep Velocity of flow in pipe CE, Veg Let the flow rate in pipe AB Velocity of flow in pipe BC. Velocity of flow in pipe o 168 Flui ‘Mechanics Diameter of pipe Then flow rate through and flow rate through (® Now volume flow rate through AB = Q = Vay x Area of AB =3.0x i (Dyg)? = 3.0 x i (1.2)? = 3.393 m/s. Ans. (ii) Applying continuity equation to pipe AB and pipe BC, Van Atea of pipe AB = Vac x Area of pipe BC Rip? X ,) 3.0 x2 Dyp)? = Voc X= (pc) or 7 Dany x5 Dror : AO." Vex 57 [ovis 2] or 1.92 mis. Ans. we TS (iii) The flow rate through pipe c= 9,2 2 = 383-131 ms 3°03 21= Ven x Area of pipe CD x (Dep)? or 1.131 = Vey og x 0.8? = 0.5026 Ven Veo = M3 = 2.25 mvs. Ans. 0.5026 (iv) Flow rate through CE, Q,= Q~ Q, = 3.393 ~ 1.131 = 2.262 mts 0s = Veg Aro pipe CE = Veg™ (De) a 2a =25 x2 x De? -VLIS2 = 1.0735 m Kinematics of Flow and Ideal Flow 169 Problem 5.4 4 25 cm diameter pipe carries oil of sp. gr. 0.9 at a velocity of 3 m/s. At another section the diameter is 20 cm. Find the velocity at this section and also mass rate of flow of oil. Solution. Given at section 1, at section 2, Mass rate of flow of oil Applying continuity equation at sections 1 and 2, AW = AV or 0.049 x 3.0 = 0.0314 x Vy V, = 20830 4.68 mus. Ans. 0.0314 Mass rate of flow of oil = Mass density x Q = p x A, x V, Density of oil Sp. ar. of oil eens of cl aC Density of water Density of o sp. gf of oll x Density of water = 0.9 x 1000 kg/m? Ns Mass rate of flow = 900 x 0.049 x 3.0 kis = 132.23 kg/s. Ans. Problem 5.5 A jet of water from a 25 mm diameter nozzle is directed vertically upwards. Assuming that the jet remains circular and neglecting any loss of energy, that will be the diameter at a point 4.5 m above the nozzle, ifthe velocity with which the jet leaves the nozzle is 12 m/s. Solution. Dia, of nozzle, Velocity of jet at nozzle, Height of point A, ero | [asm Let the velocity of the jet at a height 4.5 m= V» ware Consider the vertical motion ofthe et from the out ofthe | | nozzle to the point A (neglecting any loss of energy). D1A=25 mn Initial velocity, u=V,=12m/s ae Final velocity, Value of Using, 170 ‘Fluid Mechanics 46 mis V, =Yi2? -2x981x45 = J144— 8829 Now applying continuity equation to the outlet of nozzle and at point A, we get A\V, = AV> or AM = 2% (0.025) X12 _ 9.990796 4x 746 Let Dy = Diameter of jet at point A. Then Ay =F D," oF 0,0007896 =F x D,* (0007896 x4 © > 5.6 CONTINUITY EQUATION IN THREE-DIMENSIONS D; 0.0317 m = 1.7 mm. Ans. Consider a fluid element of lengths dr, dy and dz in the direction of x,y and z. Let u, v and w are the inlet velocity components in x, y and z directions respectively. Mass of fluid entering the face ABCD per second X Velocity in x-direction x Area of ABCD x wx (dy x de) Then miso al ing tie ZFH pe cot ut 2p Gain of mass in x-direction = Mass through ABCD ~ Mass through EFGH per second pw apc pudte- 2 pint a Sp (PH doe) de 2 podett (desma Similarly, the net gain of mass in y-direction a =- 2 (pn drdyde ay (OP a a dow) drayas a PME and in z-direction = Net gain of masses = Since the mass is neither created nor destroyed in the fluid element, the net increase of mass per unit time in the fluid element must be equal to the rate of increase of mass of fluid in the element, But mass Kinematics of Flow and Ideal Flow 171 ap Fedde Equating the two expressions, a a a ap or [ae 0+ G5 (Ov)+ 5-(0w)| dee = SP. dee 58 (ou) +? (pr) +2 (ow ancelling dx.dy.dz from both si or +5 ON) +5, (09) 45-0 = 0 [Cancelling dx.dy.dz from both sides) ...(5.34) ar Equation (5.34) is the continuity equation in cartesian co-ordinates in its most general form. This equation is applicable to = (@) Steady and unsteady flow, (ii) Uniform and non-uniform flow, and (ii) Compressible and incompressible fluids. ap For steady flow, 2? or and hence equation (5.34) becomes as a a a By 4) #35 (PP) + 5-(w) =0 (5.3B) If the fluid is incompressible, then p is constant and the above equation becomes as au, ox dy az Equation (5.4) is the continuity equation in three-dimensions, For a two-dimensional flow, the com- ae ax dy 5.6.1 Continuity Equation in Cylindrical Polar Co-ordinates. The continuity equation in Solna provers (etr hz svetinie) in devo ty prose snes Clon, Considers two dimeaional innpene low field The ‘two-dimensional polar co-ordinates are r and 8. Consider a fluid wy eae, y+ Bear element ABCD between the radii r and r + dr as shown in Cy . Fie 3.7 Theale abendooy tesla che soa ea TH conponcns of te slot Varin the ea eon and apne ange ecton The ie of he lene st faving th ape SiEAB = 8, B= dr, DC=( +47) d8, AD =r i pair same Gia cs ea om tear see Maso i atg tte AB por ani me p» Velo in rection x Area (5A) (5.5) 172. Fluid Mechanics Xu, X (AB x 1) ‘» Area = AB x Thickness = rd@ x 1) Xu, x (rd0X 1) =p + up. rd Mass of fluid leaving the face CD per unit time = px Velocity x Area = x(a) «(ed (Area = €D x1) : (a, + &. a) x (r+ dryd0 + CD = (r+ dr) d8] =p afi xr tude Se tre (ae? | a0 =o xrtu, xcdre rte ar] 40 [The term containing (dr? is very small and has been neglected] Gain of mass in r-ieetion per unit time = (Mass through AB ~ Mass through CD) per unit time = pou, 0 ole upd re | «0 pu, rd0- pu, 1 older ar] 6 2 aa " [This is written in this form because 2%] rodrod9 (7. d8. dr. 1) is equal to volume of or element] o marr Now consider the flow in @-direction Gain in mass in @-direction per unit time = (Mass through BC ~ Mass through AD) per unit time = [p x Velocity through BC x Area — p x Velocity through AD x Area] 4) arx1| + lp tro (4 + Are irx 1) [Multiplying and dividing by 7] ‘Total gain in fluid mass per u ole (3.54) Kinematics of Flow and Ideal Flow 173 But mass of fluid element = px Volume of fluid element px (rd0 x dr x1] =p xrd8.dr Rate of increase of fluid mass in the element with time a = 5, P rd8-an ( rdQ dr. Lis the volume of element and is a constant quantity) ‘Since the mass is neither created nor destroyed in the fluid element, hence net gain of mass per unit time in the fluid element must be equal to the rate of increase of mass of fluid in the element. Hence equating the wo expressions given by equations (5.5 A) and (5.5 B), we get ap SP. nto ar (5.5B) or {Cancelling rdr . d9 from both sides} or By 5.50) ale ar 690 Equation (5.5 C) is the continuity equation in polar co-ordinates for two-dimensional flow. For steady flow 2°. = 0 and hence equation (5.5 C) reduces to or a a au, or Ze +d w= o [: 2em) Bo4,] Equation (5.5 D) represents the continuity equation in polar co-ordinates for two-dimensional steady incompressible flow. Problem 5.5A Examine whether the following velocity components represent a physically possible flow ? 4, = 1 sin 8, y= 2r cos @. Solution. Given : u, = rsin @ and up = 2r cos @ For verily pase flow, the continuity equation, 2 (ru) 2 9 (to) = 0 should be satisfied. Now u,=rsin® Multiplying the above equation by r, we get ru, =P sin 174 Fluid Mechanics Differentiating the preceding equation w.r.t.r, we get a ao y (ru) 3 (F sin 8) =2rsin 8 (2 sin 0s constant watt.) Now 1g = 2r cos @ Differentiating the above equation wart. 8, we get a a 3g (te) = 5g Or eos 8) = 2r(- sin @) (eo 2r is constant w.r.t. @) 2rsin 8 a a 2 (ru) +2 (uy) = 2r sin 8 2r sin 3p ltr) 3g ) = 2r sin 8 2rsin = 0 Hence the continuity equation is satisfied, Hence the given velocity components represent a physi- cally possible flow. » 5.7 VELOCITY AND ACCELERATION Let Vis the resultant velocity at any point in a fluid flow. Let u, v and w are its component in x, y and z directions. The velocity components are functions of space-co-ordinates and time. Mathematically, the velocity components are given as, WE ACY. 2D and Resultant velocity, Let a,, a, and a, are the total acceleration in x, y and z directions respectively. Then by the chain rule of differentiation, we have dtu _u dx , au dy, Ou dz, Ou at ar But au ar Similarly, = (5.6) ae ay” a a a Kinematics of Flow and Ideal Flow 175 or ar gy By OH a, (5.1) a oe ay” a Acceleration vector (5.8) 5.7.1, Local Acceleration and Convective Accéleration. Local acceleration is defined as the rate of increase of vaey ‘with respect to time at a given point in a flow field. In the equation given as convective acceleration. Problem 5.6 The velocity vector in a fluid flow is given V = 4x'i— 10s¢yj + 2th Find the velocity and acceleration of a fluid particle at (2, 1, 3) at time t= 1. Solution. The velocity components u, v and w are w= 43°, v= ~ 10x? y, w = 2¢ For the point 2, 1, 3), we have x= 2, y= and z= 3at time t= 1. Hence velocity components at (2, 1, 3) are 4x 2)* = 32 units = 10(2)°(1) = ~ 40 units 2 units Velocity vector V at 2, 1, 3) = 321 40j + 2k s 24 or Resultant velocity = fu? +97 +? = (32? + (- 40)" + 2? = (1024+ 160044 = 51.26 units. Ans. Acceleration is given by equation (5.6) Qu au Ou, au Ret at” ta ay OF 176 Fluid Mechanics Now from velocity components, we have au _ 52, ou Mee, ar ay av oe aw ar Substituting the values, the acceleration components at (2, 1, 3) at time ¢= 1 are bo (2x2) + (— 10x"y) (0) + 2x) +0 H8x° = 48 x (2)° = 48 x 32 = 1536 units bx (— 20xy) + (= 10x°y) (— 10x”) + 2 (0) +0 = 80x4y + 100x4y = 80 (2)* (1) + 100 (2)* x 1 =~ 1280 + 1600 = 320 units. 2° (0) + C 10x*y) (0) + (2) 0) + 2.1 = 2.0 units Acceleration is Azayi+ aj + a= 15361 + 320) + 2k. Ans. or Resultant A =y(1536)° + (320) +(2)° units = 25592964 1024004 = 1568.9 units. Ans. Problem 5.7 The following cases represent the two velocity components, determine the third com- ponent of velocity such that they satisfy the continuity equation @) u=¥ey ee nye +ay (ii) v = 2y', w = aye Solution, "The continuity equation for incompressible fluid is given by equation (5.4) as, ou FS av, ow ax dy oz Case I. wereye? ayy tay Suisiting the values of and = in continuity equation. eM et dry Pere + Day rhe aw, = 3x- Day + 2 az a Kinematics of Flow and Ideal Flow 177 Integration of both sides gives [d= (3x - 2ay + 22) de (3 where constant of integration cannot be a function of z. But it can be a function of x and y that is f(x,y). ( un 2xyn+ z] +10, y). Ans, 2 av Case aye, » ay ow w= aye, ieee Substituting the values of 2 and 2 in continuity equation, we get ay a © ayer =0 ea o 4y ~ 2ay or du = 4y~ 2x9) dr Integrating, we get w= day 29> + FO.) = dy =3°y + flyy 2) Ams, Problem 5.8 4 fluid flow field is given by Vexyit yy Qye+yc)k Prove that it is a case of possible steady incompressible fluid flow. Calculate the velocity and accel- eration at the point (2, 1, 3). Solution. For the given fluid flow field « = y e 2ay ay & oo 3 = Day = yz Daye - ye? For a case of possible steady incompressible fluid flow, the continuity equation (5.4) should be satisfied. oo ou 9 Ow ~ ay a au av aw Substituting the vatues of 2, 2 ana 2, we get * ie” ay ae NB y+ Dye — Day 2ye = 0 178 Fluid Mechanics Hence the velocity field V= xvi + y*yj - Qayz + yz") kis a possible case of fluid flow. Ans. Velocity at (2, 1, 3) Substituting the values 1 and z= 3 in velocity field, we get yi + yf ~ Qaye + 92") Bx lit Px 3j-(2x2x1K341KIVK 143) 21k. Ans. and Resultant velocity = 4? +3? + C21) = J16+9 + 441 = 466 = 21.587 units. Ans. Acceleration at (2,1, 3) The acceleration components a, a, and a, for steady flow are = Dey - ye, Substituting these values in acceleration components, we get acceleration at (2, 1, 3) a= xy Qay) + y°e ( - Qayz +92") (0) 28 yay: =2QPP4PxPx3=2x8412 16 + 12.= 28 units ay = xy (0) + y"¢ Qyz)- Qrye +32) ") 292 — dry'e— ye xx 2x2 1x3 19x3? = 18- 12-9 =~3 units Py (— Dye) + ye © Bae ~ 24) - aye + ye") 2ry - 292) = eye — VP-P 4 by'y%e + Wy*? + dy? + 297] =-2x2x 1x 3-2x2xPxF- 1x3 FLAP x PB 422% 1x4 dD PT DH PD — 24 - 36 - 27 + [48 + 36 + 72 + 54] =~ 24-36-27 448 + 36+ 72 + 54 = 123 Acceleration ai + a, + a,k = 281-3) + 123k, Ans. Kinematics of Flow and Ideal Flow 179 or Resultant acceleration = 28? + (3) +123* = J784+9 + 15129 = 15022 = 126.18 units. Ans. Problem 5.9. Find the convective acceleration at the middle of a pipe which converges uniformly Srom 04 m diameter to 0.2 m diameter over 2 m length, The rate of flow is 20 lis. Ifthe rate of flow changes uniformly from 20 Us to 40 Ws in 30 seconds, find the total acceleration at the middle of the Pipe at 15th second. Solution. Given : Diameter at section 1, D, = 0.4m; Dy= 0.2 m, L= 2m, Q = 20 Ils = 0.02 m'/s as one litre 001m = 1000 em? Find (i Convective acceleration at middle ée., at A when Q= 20 lis. (ii) Total acceleration at A when Q changes from 20 U/s to 40 l/s in 30 seconds. Case I. In this case, the rate of flow is constant and equal to 0.02 m°/s. The velocity of flow is in «direction only. Hence this is one-dimensional flow and velocity components in y and z directions are zero or v= 0,2=0. au Convective aocleration = 15" only wi) Let us find the value of «and 2 a distance «from inet ex ‘The diameter (D,) ata distance x from inlet of at section X-X is given by, T rs 4 tm p,=04-" ral = (04-019 m L Leg ‘The area oferosssection (A,) at section X-X is given by, ered ton X's ra> Avs "p22" 4-019? 5 n= ED? = Ed " Fig. 58 Velocity (u) at the section X-X in terms of Q (ie in terms of rate of flow) nowt oe Area” A, i Gee = 1.273. (04-01 97? mi TE ee 1273.0 (04 - 04127? mis (i) au To find 5, we must differentiate equation (i) with respect t0 x au a . 2 namg@s-o1) 1.273 0-2) (04-01. x 0.1) [Here Q is constant] .2546 0 (0.4 - 0.1 x)! (iti) Sut Convective acceleration = [1.273 Q (0.4 ~ 0.1 2) 7] x (0.2546 Q (0.4 - 0.1 x7"] .273 x 0.2546 x Q? x (0.4 ~ 0.1 x) 180 Fluid Mechanics = 1.273 x 0.2546 x (0.02)? x 0.4 - 0.1.9 [+ Q=0.02 ms} -Convective acceleration at the middle (where x = I'm) = 1.273 x 0.2546 x (0.02)° x (0.4 = 0.1 x 1) mis? 1.273 x 0.2546 x (0.02)? x (0.3) mis? .0048 m/s. Ans. Case IL. When Q changes from 0.02 m°/s to 0.04 m°Vs in 30 seconds, find the total acceleration at x= I mand f= 15 seconds. ‘Total acceleration = Convective acceleration + Local acceleration at ¢= 15 seconds. ‘The rate of flow at r= 15 seconds is given by .04 m/s and Q, = 0.02 mls = 0) +222 515 where Q,= 0= 0 +2 TE x15 where , X15= 0.03 mis ‘au Ree eee (eee eee ec :. Convective acceleration [1.273 0 (040.1 x)71 x (0.2546 0 (0.4 ~ 0.1.39") .273 x 0.2546 Q? x (0.4 ~ 0.1 x 1)* ++ Convective acceleration (when Q = 0.03 m’/s and.x= I m) 1.273 x 0.2546 x (0.03)° x (0.4 - 0.1 1)? 273 x 0.2546 x (0.03)* x (0.3)* m/s* .0108 mis? liv) Bes ar 11.273 90.40.17] [+s 4 from equation (i) is w= 1.273 Q (0.4 -0.1.x)] a Local aceeleration = 57 sia xco4 ony 22 [ +» Local acceleration is at a point where x is constant but Q is changing] Local acceleration (at x = 1 m) 1273 x (04-04 x 12x 22 ar Vice tot-o12 om xox at-00t 22) = 0.00943 m/s” (vy) Hence adding equations (iv) and (1), we get total acceleration. <- Total acceleration = Convective acceleration + Local acceleration 10108 + 0.00943 = 0.02023 mis’, Ans. Kinematics of Flow and Ideal Flow 181. > 5.8 VELOCITY POTENTIAL FUNCTION AND STREAM FUNCTION 5.8.1 Velocity Potential Function. It is defined as a scalar function of space and time such that its negative derivative with respect to any direction gives the fluid velocity in that direction. Itis defined by 6 (Phi). Mathematically, the velocity, potential is defined as @ = f(x,y, 2) for steady flow such that (5.9) where u, v and w are the components of velocity in x, y and z directions respectively, ‘The velocity components in cylindrical polar co-ordinates in terms of velocity potential function are given by 2 (594) Where u,= velocity component in radial direction (i.e. in r direction) and ug= velocity component in tangential direction (i.e. in @ direction) du | av | ow “The continuity equation for an incompressible steady flow is 4 4 9 4 OY ax ay ae Substituting the values of u, v and w from equation (5.9), we get 2 (_a), 2 (_a 2 *) 2/-2).2 +2(-2) <0 x 3) xl ay)” aa ao , a aierneie) 5.10 or te (5.10) Equation (5.10) is a Laplace equation. Ho, (5.11) For two-dimension case, equation (5.10) reduces to SF +5 ¢ If any value of 6 that satisfies the Laplace equation, will correspond to some case of fluid flow. Properties of the Potential Function. The rotational components* are given by G3) * Please, refer to equation (5.17) on page 192. 182 Fluid Mechanics 1% =) Tae ae 1 (aa ay a Substituting the values, of u, v and w from equation (5.9) in the above rotational components, we get 26 2 2) af oo , ae dy) ay ar. 2| axdy dyax ( 2) Ft ox. Oxdz and 2 *) ae * 2Lay ae ded 114 18 a cominvous function, hen 2-# = 2 ay When rotational components are zero, the flow is called irrotational. Hence the properties of the potential function are : 1. If velocity potential (6 exists, the flow should be irrotational 2. If velocity potential (6) satisfies the Laplace equation, it represents the possible steady incom- pressible irotationa flow 5.8.2 Stream Function. It is defined asthe scalar function of space and time, such that its partial derivative with respect to any direction gives the velocity component aright angles to that direction. It is denoted by y (Psi) and defined only for two-dimensional flow. Mathematically, for steady flow itis defined as y= f(t 9) such that ow ox (5.12) and sou ay The velocity components in cylindrical polar co-ordinates in terms of stream function are given as Lay ay 2M and uy = - 2 5.124 ra ar came where u,= radial velocity and ug = tangential velocity Kinematics of Flow and Ideal Flow 183 Substituting the values of w and v from equation (5.12), we get a (_2v), 2/2) og Sv, Hy xl ) 5 (B)-0% Didy * dxdy ay Hence existence of y means a possible case of fluid flow. The flow may be rotational or irrotational. 1(av_aw ‘The rotational component «, is given by @, = —| 1 is given by ©, 1(% x) Substituting the values of w and y from equation (5.12) in the above rotational component, we get For irrotational flow, @, = 0. Hence above equation becomes as2¥ which is Laplace equation for y. The properties of stream function (y) are = 1. If stream function (y) exists, it is a possible case of fluid flow which may be rotational or irrotational. 2. If stream function (y) satisfies the Laplace equation, itis a possible case ofan irotational flow. 5.8.3 Equipotential Line. A line along which the velocity potential @ is constant, is called cequipotential line ‘ dy = ace ay do = ae dx + ay dy 20 (ude + vdy). For equipotential line, or ~ (uds + vy) aw ae dy av ine of Constant Stream Function = Constant day or udx + vdy = 0 (5.13) But Slope of equipotential line. But d+ ay = 4 vde—udy ay 184. Fluid Mechanics For a line of constant stream function = dy = 0 or vdx~udy = 0 ay acu or (5.14) bur soe ot tam ne, From equations (5.13) and (5.14) itis clear that the product of the slope of the equipotential line and the slope of the stream line at the point of intersection is equal to ~ 1. Thus the equipotential lines are ‘othogonal to the stream lines at all points of intersection. 5.8.5 Flow Net. A grid obtained by drawing a series of equipotential lines and stam lines iscalled flow net. The flow net is an important tool in analysing two-dimensional rotational flow problems. 5.8.6 Relation between Stream Function and Velocity Potential Function From equation (5.9), we have “ From equation (5.12), we have u oy Thus, we have u x ar 9% Hence a ar 5.15) and 3 Problem 5.10 The velocity potential function (6) is given by an expression woe 3 3 (i), Find the velocity components in x and y direction. (ii) Show that 6 represents a possible case of flow. we Pry Solution. Given : =a Ae Gi oe AD ay The partial derivatives of @ wart. x and y are +3 ay and. +2y 2) Kinematics of Flow and Ideal Flow 185 (® The velocity components u and v are given by equation (5.9) From equations (1) and (2), we have ci Now By and ca) ay ar 24 day Day +2 Ans. YP - 204% wien yr 4 2y 24 2y) + 2y+2)=0 Laplace equation is satisfied and hence @ represent a possible case of flow. Ans. Problem 5.11 The velocity potential function is given by 6 = 5 (x? y*). Calculate the velocity components at the point (4, 5). Solution. 6250-97) But velocity components u and v are given by equation (5.9) as oa 26 10x © ly) = toy ‘The velocity components at the point (4, 5), ie., atx=4, y= 5) ~10x4 10x 40 units. Ans. 30 units. Ans. 186 Fluid Mechanics Problem 5.12 A stream function is given by y = 5x —6y. Calculate the velocity components and also magnitude and direction of the resultant velocity at any point Solution. lu? +? = 6 +5? = f36+25 =V6T = 7.81 univisee Resultant velocity vs Direction is given by, tan @ j= 0.833 Ei = tan! 833 = 39° 48’. Ans. Problem 5.13 If for a mwo-dimensional potential flow, the velocity potential is given by ey) determine the velocity at the point P (4, 5). Determine also the value of stream function W at the point P. 9=xQy-1) Solution. Given : (@ The velocity components in the direction of x and y are 9 units/sec = 8 units/sec Velocity at P 9i Resultant velocity at P= 9? +8° = JBI+64 = 12.04 units/sec = 12.04 units/sec. (id) Value of Stream Funetion at P A) Atthe point P (4,5), ie., atx Ans. We know that ii) and. y's we get Integrating equation () w.r. Jdy = J (2y-1)dy or y= Constant of integration. Kinematics of Flow and Ideal Flow 187 ‘The constant of integration is not a function of y but it can be a function of x. Let the value of ‘constant of integration is k. Then way -ytk iii) Differentiating the above equation wars. ‘x, we get oy ax av But from equation (i), SY =~ 2x t from equation (i), SY Integrating this equation, we get k= J ~ 2d Substituting this value of k in equation (i), we get y= y?=y =x. Ams. ++ Stream function y at P (4, 5) = 5-5-4? = 25 - 5-16 = 4 units. Ans. Problem 5.14 The stream function for a two-dimensional flow is given by y = 2xy, calculate the velocity atthe point P (2, 3). Find the velocity potential function 6. Solution. Given > wey ‘The velocity component and vin terms of y are aya 22 @ ay ay ay_a v= 22 ayy 22, or aS ry) = 2y, Atte point P (2,3), we get w= 22 = —4 unitsee ye 2x3 = Ounitlsee Resultant velocity at P= Yu? +v? = Ja? +6 = f16436 = VSD = 7.21 unitsisec. Velocity Potential Funetion We know u=-( 2228 @ -2y ii) Integrating equation (9, we get Jodo =) 2xax 2x 2 = pcevec or o=acete where C is a constant which is independent of x but can be a function of y. Differentiating equation (i wrt. y', we get 22 = 9 a ay 188 Fluid Mechanics But from (ii), Integrating this equation, we get C= [- 2y dy ‘Substituting this value of C in equation (ii), we get @ = x?—y7, Ans. Problem 5.15. Sketch the stream lines represented by Y Also find out the velocity and its direction at point (1, 2). Solution. Given yer? ‘The velocity components 1 and v are 2y 2 m~ 4 UNITS /SEC At the point (1, 2), the velocity components are 2x 1=2 units/sec Resultant velocity = fi ty? = (4) +2? ey =V20 = 4.47 units/see 1 and, tan =~ u ud 2 x @= tan 5 = 26° 34" Resultant velocity makes an angle of 26° 34” with x-axis, Fig. 5.10 = 2 2=-4 unitssec i Sketch of Stream Lines versy? Let = 1, 2,3 and so on. ‘Then we have l=e4y? eter 3erey and so on, Each equation is a equation of a circle. Thus we shall get concentric circles of different diameters as shown in Fig. 5.10. Problem 5.16 The velocity components in a two-dimensional flow field for an incompressible fluid are as follows : we 2 gre eyandve ny? 29-28 obtain an expression for the stream function Y. Kinematics of Flow and Ideal Flow 189 Solution. Given : ua y+ 2e-ey veo? 2y- 88. The velocity components in terms of stream function are ov M eye nt 2-08 0) OY aya y9-204%y Aid) ay Integrating (f) wart. x, we get y= J (9? = 2y 18) de where is a constant of integration which is independent of x but can be a function of y. Differentiating equation (iii) w.t. y, we get But from (ii), Comparing the value of 2¥., we get x oy ak ay YB 244.2 Ineraing, we st k= fe Pay Substituting this value in (ii), we get Problem 5.17 ina two-dimensional incompressible flow, the fluid velocity components are given by u=x-—dyandy =-y-4n. Show that velocity potential exists and determine its form. Find also the stream function. Solution. Given : wex-4y and au ox au, ox ay Hence flow is continuous and velocity potential exists. Let = Velocity potential. 190 Fluid Mechanics Let velocity components in terms of velocity potential is given by ws —(x-4y) x+dy Ai) and. ve-Gy-4nay44e ii) 2 Integrating equation (9), we get 9 =~ + day-+ eoliii) where C is a constant of integration, which is independent of x. This constant can be a function of y. Differentiating the above equation, i.e., equation (iii) with respect to ‘y’, we get co ay ac oars a : 2% Equating the two values of 2° , we get ay ye t4ay +2. Ans. aa) Value of Stream functions Let y = Stream function The velocity components in terms of stream function are ~y-4e Gv) and ow = 4y) 2-4 4y ol) wr 4) iy “ Integrating equation (7) wats. x, we get 42 2 ak (oi veo «wv, where k is a constant of integration which is independent of x but can be a function of y. Kinematics of Flow and Ideal Flow 191 Differentiating equation (vt) was. y, we get 2 =— 2-04 2k ay oy oy Bat from equation (»), we have aw it -quation (v), ay oy ak Bquating the two vatuesof 2%, we get — 242% ay ay 2 Integrating the above equation, we get ae ay? Substituting the value of k in equation (vi), we get yenyx 2x? + 2y7, Ans. > 5.9 TYPES OF MOTION A fluid particle while moving may undergo anyone or combination of following four types of displacements : (®) Linear Translation or Pure Translation, (ii) Linear Deformation, (iii) Angular Deformation, and (iv) Rotation. 5.9.1 Linear Translation. It is defined as the movement of a fluid element in such a way that it moves bodily from one position to another position and the two axes ab and cd represented in new positions by a’b and c’d” are parallel as shown in Fig. 5.11 (a) 5.9.2 Linear Deformation. It is defined as the deformation of a fluid element in linear direction when the element moves. The axes of the clement in the deformed position and un-deformed position are parallel, but their lengths change as shown in Fig. 5.11 (2). y « fe ; «Poy Se 4 Tle sy op = x x (0) LUNEAR TRANELATION = Bes x % (@) ANGULAR DEFORMATION (@) PURE ROTATION Fig. 5.11, Displacement of a fluid element. 192. Fluid Mechanics 5.9.3 Angular Deformation or Shear Deformation. It is defined as the average change in the angle contained by two adjacent sides. Let A@, and AQ, is the change in angle between two adjacent sides of a fluid element as shown in Fig. 5.11 (c), then angular deformation or shear strain rate 1 = 0, + 28, ze + A8,) Now 6, =H 5, At ay Ax ax ‘Angular deformation = + [A8, + A8,] 1fav. aw Shear strain rate = 4/2" (5.16) ce serait a2 3] 5.9.4 Rotation. It is defined as the movement of a fluid element in such a way that both of its axes (horizontal as well as vertical) rotate in the same direction as shown in Fig. 5.11 (d). It is equal 1 (2 ou ax ay (S17) 5.9.5 Vorticity. It is defined as the value twice of the rotation and hence it is given as 20 Problem 5.18 A fluid flow is given by V = 8xi~ 10x". Find the shear strain rate and state whether the flow is rotational or irrotational.. Solution. Given : Vs 8x'i- 10x°yj au ou u = 2407, Br, ro im, yy 2 ov and v= = 1Or'y, 5 = = 2 (@) Shear strain rate is given by equation (5.16) as (& [ Kinematics of Flow and Ideal Flow _193 | (i) Rotation in x - y plane is given by equation (5.17) or (2 *) * 2029-0) == 1039 2 lax ay) 2 As rotation ©, # 0. Hence flow is rotational. Ans. Problem 5.19 The velocity components in a two-dimensional flow are w= + 2x —27y and v = xy? ~2y- 0B. Show that these components represent a possible case of an irrotational flow. Solution. Given : us yB+2x- xy au ar (®)_ For a two-dimensional flow, continuity equation is 2 au Substituting the value of “ and 2, we get ituting Fe am Sr me get au , av BH 8 2 ay 4 2-2-0 ax ay ceded tis a possible case of fluid flow. 1fdv du ii) Rotation, @, is given by @, = +{ 2-2 (i) Rotation, @, is given by ©, (2 = Rotation is zero, which means it is case of irrotational flow. Ans. =1w-_2-G-2= ) 7 WO? -7)- 07-1 =0 > 5.10 VORTEX FLOW ‘Vortex flow is defined as the flow of a fluid along a curved path or the flow of a rotating mass of fluid is known a ‘Vortex Flow’. The vortex flow is of two types namely : 1, Forced vortex flow, and 2. Free vortex flow. 5.10.1 Forced Vortex Flow. Forced vortex flow is defined as that type of vortex flow, in which some external torque is required to rotate the fluid mass. The fluid mass in this type of flow, rotates at constant angular velocity, «. The tangential velocity of any fluid particle is given by veoxr (5.18) [194 Fluid Mechanics ] where r= Radius of fluid particle from the axis of rotation. | CENTRAL AXIS Leu [i fom (8) CYLINDER IS STATIONARY (b) CYLINDER IS ROTATING VERTICAL-"} CYLINDER, Fig.5.12. Forced vortex flow. Hence angular velocity @ is given by © Constant. (5.19) Examples of forced vortex are : 1. A vertical cylinder containing liquid which is rotated about its central axis with a constant angular velocity (as shown in Fig. 5.12. 2. Flow of liquid inside the impeller of a centrifugal pump. 3. Flow of water through the runner of a turbine. 5.10.2 Free Vortex Flow. When no external torque is required to rotate the fluid mass, that type of flow is called free vortex flow. Thus the liquid in case of free vortex is rotating due to the rotation which is imparted to the fluid previously. Examples of the free vortex flow are : 1. Flow of liquid through a hole provided at the bottom of a container. 2. Flow of liquid around a citcular bend in a pipe. 3. A whirlpool in a river. 4, Flow of fluid in a centrifugal pump casing. ‘The relation between velocity and radius, in free vortex is obtained by putting the value of external torque equal to zero, or, the time rate of change of angular momentum, i.e., moment of momentum must be zero. Consider a fluid particle of mass ‘m’ at a radial distance r from the axis of rotation, hhaving a tangential velocity v. Then ‘Angular momentum = Mass x Velocity =m xv Moment of momentum Momentum x r= m xv xr a ‘Time rate of change of angular momentum = o (mvr) a For free vortex 2 (myn ar Constant Integrating, we get mr = Constant or vr = S49" = Constant (5.20) [ Kinematics of Flow and Ideal Flow _ 195] 5.10.3 Equation of Motion for Vortex Flow. Consider a fluid element ABCD (shown shaded) in Fig. 5.13 rotating at a uniform velocity in a horizontal plane about an axis perpendicular to the plane of paper and passing through 0. Let r= Radius of the element from 0. A@ = Angle subtended by the element at 0. ‘Ar = Radial thickness of the element. AA = Arca of cross-section of element. ‘The forces acting on the element are : (@) Pressure force, pAA, on the face AB. (iid) Centrifugal force,"""— acting in the direction away from the centre, 0. Now, the mass of the element: Mass density x Volume x AA x Ar Centrifugal force = pAddr Equating the forces in the radial direction, we get ar (rt) a4 pat ptr : 2 are patar’ necting Ar AA from bah sides, we get = 9 (5.21) Equation (5.21) gives the pressure variation along the radial direction for a forced or free vortex flow na horizontal plane. The expression 2 is called pressure gradient inthe rail direction, AsSE is positive, hence pressure increases with the increase of radius *r. The pressure variation inthe vertical plane is given by the hydrostatic law, i, ap a In equation (5.22), z is measured vertically in the upward direction. ‘The pressure, p varies with respect tor and zor p is a function of r and z and hence total derivative ofpis Pe (5.22) pe Bare, Substituting the values of 2 from equation (5.21) and 22 from equation (5.22), we get ar a2 [196 Fluid Mechanics ] dp = p~— dr~ pad (5.23) Equation (5.23) gives the variation of pressure of a rotating fluid in any plane, 5.10.4 Equation of Forced Vortex Flow. For the forced vortex flow, from equation (5.18), we have veoxr where © = Angular velocity = Constant. Substituting the value of v in equation (5.23), we get oF dp =p x2 dr— pg de. Consider two points 1 and 2 in the fluid having forced vortex flow as shown in Fig. 5.14. Integrating the above equation for points 1 and 2, we get fin = f ouvir Fonte or @-P)= [ro =] - pe [cht or @2~ p= PF tr? = nl 98 f= 21 § long — 0r)'1- pg lesa) 2 son = 2 be —w?l- pg tesa { } If the points 1 and 2 lie on the free surface of the liquid, then p) hence above equation becomes 0 or ps [z~ 211 ~2)= = f2-», or Fan al= 55 ba - nih If the point I lies on the axis of rotation, then v, = @ x r, = @ x 0= 0. The above ‘equation becomes as nk Let act (5.24) [ Kinematics of Flow and Ideal Flow _197| ‘Thus Z varies with the square of r. Hence equation (5.24) is an equation of parabola. This means the free surface of the liquid is a paraboloid, Problem 5.20 Prove that in case of forced vortex, the rise of liquid level at the ends is equal to the fall of liquid level at the axis of rotation. Solution. Let R= radius of the cylinder. (0-0 = Initial level of liquid in cylinder when the cylinder is not rotating. - Initial height of liquid hen) <. Volume of liquid in cylinder = mR? x Height of liquid AR? x (+3) i) Let the cylinder is rotated at constant angular velocity (. The liquid will rise at the ends and will all at the centre. Let Rise of liquid at the ends from 0-0 xr Fall of liquid at the centre from 0-0. Then volume of liquid = [Volume of cylinder upto level B-B] = [Volume of paraboloid] = [RR? x Height of liquid upto level B-B] RR ~ |=» Height of paraboloid eax (he x+y) ee xray) ikon wexneaecsy xen : 2 mR? 2 ety Ai’) Equating (i) and (ii), we get BP th x)= wR EE xe y) or or Fall of liquid at centre = Rise of liquid at the ends. Problem 5.21 An open circular tank of 20 cm diameter and 100 cm long contains water upto a height of 60 cm. The tank is rotated about its vertical axis at 300 r.p.m., find the depth of parabola formed at the free surface of water. Solution, Given : Diameter of cylinder = 20m 20 Radius, R= = 10cm 7710 [198 Fluid Mechanics ] Height of liquid, Speed, ‘Angular velocity, = 2XEX300 «5141 radiecc, 0 Let the depth of parabola Using equation (5.24), = (oy 50.28 cm. Ans. 2g 2x 981 Problem 5.22 An open circular cylinder of 15 cm diameter and 100 cm long contains water upto 4@ height of 80 cm, Find the maximum speed at which the cylinder is to be rotated about its vertical axis $0 that no water spills. Solution. Given : Diameter of cylinder = 15m + Radius, Length of cylinder, Initial height of water Let the cylinder is rotated at an angular speed of « rad/sec, when the water is about to spill. Then using, Rise of liquid at ends Fall of liquid at centre But rise of liquid at ends cength ~ Initial height = 100 ~ 80 = 20 em Fall of liquid at centre = 20 em Height of parabola 20 + 20 = 40 em Z=40 om oR Using the relation, Zz we get 49 = 279) v raid 2981 402X981 _ 1595 75x75 = 13952 = 37.35 rad/s 2aN «. Speed, Nis given by @ = “2. Ps given by a5 or 60x@ _ 603735 2n (xR Problem 5.23 A cylindrical vessel 12 cm in diameter and 30 cm deep is filled with water upto the top. The vessel is open at the top. Find the quantity of liquid left in the vessel, when itis rotated about its vertical axis with a speed of (a) 3000 r.p.m., and (b) 600 r.p.m. Solution. Given : Diameter of cylinder = em Radius, R=60m Initial height of water = 30cm [ Kinematics of Flow and Ideal Flow _199 | Initial volume of water = Area x Initial height of water = Fi x 12? x 30 cm? = 3392.9 em’ (a) Speed, @?R? _ (3141)? x6? 2g 2x981 Height of parabola is given by Z= = 18:10 em, As vessel is initially full of water, water will be spilled if itis rotated. Volume of water spilled is ‘equal to the volume of paraboloid. But volume of paraboloid = [Area of cross-section x Height of parabola] + 2 DP xZ =F x 12? x!810 | 1023.53 cm? 2°4 2 Volume of water left tial volume ~ Volume of water spilled 3392.9 - 1023.53 = 2369.37 em’. Ans. (b) Speed, N= 600 rpm. BAN _ 28600 _ 69.89 rads 0 0 Height of parabola, 720. C8) x6 a 40m, 2g 2x981 [As the height of parabola is more than the height of cylinder the shape of imaginary parabola will be as shown in Fig. 5.17. Let r= Radius of the parabola at the bottom of the vessel. Height of imaginary parabola = 72.40 ~ 30 = 42.40 em. Volume of water left in the vessel ‘olume of water in portions ABC and DEF = Initial volume of water = Volume of paraboloid AOF + Volume of paraboloid COD. Now volume of paraboloid AOF = © x D? x Height of parabola PA" = 4094.12 em? macitiary " maciary 2 CYLINDER’ PARABOLA For the imaginary parabola (COD), « = 62.82 rad/see Fig. 5:17 Z= 424 om Radius at the bottom of vessel [200 Fluid Mechanics ] oF 62.82? xr? ising the relation Zz we get 424 = 9282 xr eee 2g | Ne Bt 2981 ; 9814240 _ 91 79 62.82 x 62.82 r= Y2L079 = 4.59 em ‘Volume of paraboloid COD = J Area athe top ofthe imaginary parabola x Height of parabola 1 ex 4.59? x 42.4 = 1403.89 cm? adn x ard 2 2 Volume of water left = 3392.9 ~ 4094.12 + 1403.89 = 702.67 em*. Ans. Problem 5.24 An open circular cylinder of 15 cm diameter and 100 cm long contains water upto a height of 70 cm. Find the speed at which the cylinder is to be rotated about its vertical axis, so that the axial depth becomes zero. Solution, Given : Diameter of cylinder Radius, Length of cylinder Initial height of water = 70 em. ‘When axial depth is zero, the depth of paraboloid = 100 em. be 18cm —1 Fig. 5.18, Using the relation, 2nH - Speed, Nis given by @ = == ps given by 7 or N= DEO _ 10%5905 «563.98 rpm, Ans. 2 2n Problem 5.25 For the problem (5.24), find the difference in total pressure force (i) at the bottom of cylinder, and (ii) at the sides of the cylinder due to rotation. [ Kinematics of Flow and Ideal Flow _201| Solution. (i) The data is given in Problem 5.24. The difference in total pressure force at the bottom of cylinder is obtained by finding total hydrostatic force at the bottom before rotation and after rotation. Before rotation, force = pg x (0.15)? m?, hi = 70 em = 0.70 m where p= 1000 kg/m’, A = Area of bottom => D’ a Force = 1000 x 9.81 x5 x (0.15)* 0.7 N = 121.35 N After rotation, the depth of water at the bottom is not constant and hence pressure force due to the height of water, will not be constant. Consider a circular ring of radius r and width dr as shown in Fig. 5.19. Let the height of water from the bottom of the tank upto free surface of water at a radius Hydrostatic force on ring at the bottom, AP = pg x Area of ring x Z = 1000 x 9.81 x 2nrdr x © or? 28 9810 x 2.x Rr x xdr Total pressure force at the bottom wy afar = [f 9810x2285 ons =f waco tr a Fig. 519 From Problem $24, = 59.05 rads R75 0m= 075 m. Substituting these values, we get ftal pressure foree __ 196201 x (59.057 [<} 2x981 «| 4 |, 19620 x x x(59.05)* (075) _ 6.69 2x981 4 Difference in pressure forces at the bottom 121.35 ~ 86.62 = 34,73 N. Ans. (i) Forces on the sides of the cylinder Before rotation peal where A'= Surface area ofthe sides ofthe cylinder upto height of water RD x Height of water = m x .15 x 0.70 m? = 0.33 m? [202 Fluid Mechanics ] Ji = CG. of the wetted area of the sides 1 0.70 5 x height of water = > 3 * helaht of water = “> 135m Force on the sides before rotation = 1000 x 9.81 x 0.33 x 0.35 = 1133 N After rotation, the water is upto the top of the cylinder and hence force on the sides 1000981 «Wot ae of te sis High of wa B10 x Dx 10% Ex 1.0= 9810 mx 1S x 5 = 231143 N Difference in pressure on the sides 2311.43 ~ 1133 = 1178.43 N. Ans. 5.10.5 Closed Cylindrical Vessels. If a cylindrical vessel is closed at the top, which contains some liquid, the shape of paraboloid formed due to rotation of the vessel will be as shown in Fig. 5.20 for different speed of rotations. Fig. 5.20 (a) shows the initial stage of the cylinder, when it is not rotated. Fig. 5.20 (b) shows the shape of the paraboloid formed when the speed of rotation is (. Ifthe speed is increased further say , the shape of paraboloid formed will be as shown in Fig. 5.20 (c). In this case the radius of the parabola at the top of the vessel is unknown. Also the height of the paraboloid formed corresponding to angular speed @ is unknown. Thus to solve the two unknown, we should have two equations. One ‘equation is 28 ‘The second equation is obtained from the fact that for closed vessel, volume of air before rotation is equal to the volume of air after rotation, ‘Volume of air before rotation = Volume of closed vessel ~ Volume of liquid in vessel Zz me xZ 2 J T I ‘Volume of air after rotation = Volume of paraboloid formed 1 »? I i @ © ° Fig. 5.20, Problem 5.26 A vessel, cylindrical in shape and closed at the top and bottom, contains water upto 4 height of 80 cm. The diameter of the vessel is 20 cm and length of vessel is 120 cm. The vessel is rotated at a speed of 400 r.p.m. about its vertical axis. Find the height of paraboloid formed. [ Kinematics of Flow and Ideal Flow _203 | Solution. Given : Initial height of water = 80 em Diameter of vessel = 20cm v. Radius, R= 10cm Length of vessel = 120 em 7 Speed, N= 400 rpm. mou 2AM 28400 - 41.88 reais «60 When the vessel is rotated, let Z Height of paraboloid formed no 1r= Radius of paraboloid at the top of the vessel This is the case of closed vessel Fig. 521 ‘Volume of air before rotation = Volume of air after rotation or 2 ptxi-" ptxso=nPxZ 4 4 2 where Z = Height of paraboloid, r= Radius of parabola. or © px 120- © p?x s0= nex Z 4 4 2 or xD? (120 ~ 80) = FD? x40 Rag? Zz or = x 20? x 40 = 4000 x w= mr x = F 20° x 40 = 4000 x x= 7 x S Pxz = ROXRX? _ s009 ld) ® a 4188 x7? _ 4188 x? - Using relation z we get z= SBS 0° MAB 7" _ 9.504 1 * 2 8 28 2xo81 7 OS z be 0804 Substituting this value of rin (), we get 2 x 7= 3000 O08 B = 8000 x 0.894 = 7152 Zea =tasoomam aw | Hind Method 0 Let Z, = Height of paraboloid, ifthe vessel would not have been | ‘closed at the top, corresponding to speed, N= 400 rpm, or = 41.88 rad/s PR? _ 4188? x10? 89.34 cm. 26 2x981 Then zy [204 Fluid Mechanics ] Half of Z, will be below the inital height of water in the vessel 2 _ 3934 A a But height of paraboloid for closed vessel = CO = CA + AO = (120 ~ 80) + 44.67 em 40 + 44.67 = 84.67 cm. Ans. Problem 5.27 For the data given in Problem 5.26, find the speed of rotation of the vessel, when axial depth of water is zero. Solution, Gen ry ie Ao = 44.67 om Diameter of vessel 20 em +. Radius, R= 10cm Initial height of water 80 cm T ‘so Length of vessel = 120m 120 Let ais the angular speed, when axial depth is zero ao When axial depth is zero, the height of paraboloid is 120 em and +. Using the relation, 129= 2% %980 oP =2 x 980 x 120 = 235200 of) ‘Volume of air before rotation = Volume of air after paraboloid mR? x (120 ~ 180) = Volume of paraboloid Baz enexZ 2 or x10? «40 = =x 120 x10? x40x2_ 8000 or Pe RXNO X40? _ 2000 «65.67 x 120 120 Substituting the value of ? in equation (i), we get @? x 66.67 = 235200 59.4 rads 6667 2nN 2 Speed Nis given by @ = 2. P aiven by @= or N= BO OO%504 _ 567.22 ppm. Ans, 2 aR Problem 5.28 The cylindrical vessel of the problem 5.26 is rotated at 700 r.p.m. about its vertical axis. Find the area uncovered at the bottom of the tank: Solution. Given Initial height of water = 80 cm Diameter of vessel 20 em e Radius, 10 em Length of vessel = 120 em [ Kinematics of Flow and Ideal Flow _205 | Speed, N= 700 rpm. PRN _ 228X700 _ 73.30 rads, 0 0 If the tank is not closed at the top and also is very long, then the height of parabola corresponding to @= 733 will be oP xR? _ 7337 x10? 2xg 2980 From Fig. 5.24, 4) + 120 +x, = 274.12 or xy + y= 274.12 ~ 120 = 154.12 em ...() From the parabola, KOM, we have oe _ B3? x4 120 +x) = 2 = eli cs » 2g 2x 980 Se For the parabola, LON, we have Lal * 208, BR xe , ef 7 2g 2x 980 . Fig. 5.24 Now, volume of air before rotation = Volume of air after rotation Volume of air before rotation = nR* x (120 ~ 80) = x x 10° x 40 = 1256.3 em* =i) Volume of air after rotation = Volume of paraboloid KOM ~ volume of paraboloid LON cnr (BEA) gee AL © Equating (iv) and (0), we get m2 (104%) _ efx mit a) _ alc Substituting the value of r,? from (ii) in (vi), we get (120+ x,) «2980 7 1256.3 = =i) (204%) _ nex 2 2980 x (120 + x (733) 12566.3 = x x or 12566.3 = 0.573 (120 + x)? Substituting the value of x from (iff) in the above equation 33? xr J 2 “2980 12566.3 = 0.573 | 120+ 2980 0.573 (120 + 2.74 133-43 x rx ry? = 0.573 [120° + 2.74? r+ 2 120 x 2.74 re] 43 ry [206 Fluid Mechanics = 0.573 [14400 + 7.506 r,! + 657.6 73°] — 4.3 1 125663 21930 = 14400 + 7.506 r,* + 657.6 r;° 4.3 734 0573, or 13! (7.506 ~ 4.3) + 657.6 r,° + 14400 - 21930 = 0 or 3.206 rf + 657.6 r,? ~ 7530 =0 + 657.6" —4 x(~7530) x (3206) > _ 657. 23206 657.6 + [432A37.16 + 96504.72 6a = 9516S 2732 _ _ 915.98 of 10.87 o4l2 Negative value is not possible 2 rp = 10.87 em? +. Area uncovered at the base = mr,? = x 10.87 = 34.149 em®, Ans. Problem 5.29 A closed cylindrical vessel of diameter 30 cm and height 100 cm contains water upto a depth of 80 cm. The air above the water surface is at a pressure of 5.886 N/cm?. The vessel is rotated at a speed of 250 r.p.m. about its vertical axis. Find the pressure head at the bottom of the vessel : (a) at the centre, and (b) at the edge. Solution, Given : Diameter of vessel = 30cm v. Radius, R= 150m Initial height of water, H/= 80 em Length of cylinder, L= 100em Pressure of air above water = 5.886 Niem? or p= 5.886 x 10'S Head due to pressure, h= ppg = 5886%10" 6 m of water 1000x9381 Speed, N= 250 rpm. 2nN _ 2nx250 =e 26.18 rads oO 60 Height of paraboloid formed, if the vessel is assumed open at the top and it is very long. y°R? _ 26.18" x15 2g 2x 981 Let r; is the radius of the actual parabola of height x2 Then we have a = 78.60 cm oo) [ Kinematics of Flow and Ideal Flow 207 | o'r? _ 2618? x4 Then x 35, di 23g 7 a qog1 O57 @ ‘The volume of sir before rotation RR? (100 ~ 80) = wx 15? x 20 = 14137 em? Volume of air after rotation = Volume of paraboloid EOF 1 =p xan But volume of air before and after rotation is same. : 14137 = bx are But from (i), 43 £ 0.35 7)? 14137 = 4 x nr? 035 1? pte RAMS Sosa #x035 r= 25714)" = 12.66 cm Substituting the value of r, in (i), we get 2) = 0.35 x 12.66? = 56.1 em Pressure head at the bottom of the vessel (2) At the centre. The pressure head at the centre, -e., at H = Pressure head due to air + OH .0 + (HL~ LO) (OH = LH- Lo} + HL=1000m=1m = 60 + (1.0 0.561 soeda-assy fore Scien sétal (b) At the edge, i.e., at G = Pressure head due to air + height of water above G soracesos (GM Ht) =60 + G03) om it~ o00s6 { risemceasn | = 6.0 + 0.439 + 0.786 = 7.225 m of water, Ans. Problem 5.30 A closed cylinder of radius R and height H is completely filled with water. It is rotated about its vertical axis with a speed of @ radians/s. Determine the total pressure exerted by water on the top and bottom of the cylinder. Solution. Given Radius of cylinder =R Height of eylinder =H ‘Angular speed =o [208 Fluid Mechanics ] AS the cylinder is closed and completely filled with water, the rise of water level at the ends and depression of water at the centre due to rotation of the vessel, will be prevented. Thus the water will exert force on the complete top of the vessel. Also the pressure will be exerted at the bottom of the cylinder. te Rf ‘Total Pressure exerted on the top of cylinder. The top of cylinder is in contact with water and is in horizontal plane. The pressure variation at any radius in horizontal plane is given by equation (5.21) D or ae Integrating, we get Jap =Spotrdr or p= Consider an elementary circular ring of radius r and width dr on the top of the cylinder as shown in Fig. 5.26. ‘Area of circular ring = 2nrdr Force on the elementary ring = Intensity of pressure x Area of ring Fig. 5.26 = px 2nrdr x 2nrdr. ® wr} & a'r? x amr { e Total force on the top of the cylinder is obtained by integrating the above equation between the limits 0 and R. Toa fore or Fy= [ 207s? xanntr= Be? x2K f" Par 6.25) Total pressure force on the bottom of cylinder, Fy = Weight of water in cylinder + total force on the top of cylinder 1g x MR x H+ Fy (3.26) pgx nex H+ ob x er p= Density of water. Problem 5.31 A closed cylinder of diameter 200 mm and height 150 mm is completely flled with water. Calculate the total pressure force exerted by water on the top and bottom of the cylinder, if it is rotated about its vertical axis at 200 r-p.m. Solution. Given Dia. of cylinder = 200 mm = 0.20 m Radius, R=01m Height of cylinder, H= 150 mm = 0.15 m Speed, N= 200 rp.m, 2nN _ 2nx200 Angular speed, ea) ues oo 60 [ Kinematics of Flow and Ideal Flow 209 | Total pressure force on the top of the cylinder is given by equation (5.25) y= Bat cca « 120 52908 52% 0." SAME N An 4 Now total pressure force on the bottom of the cylinder is given by equation (5.26) as F y= pg x mk? x H+ Fy 1000 x 9.81 x mx (OI)? x 0.15 + 34.44 = 46.22 + 34.44 = 80.66 N. Ans. 5.10.6 Equation of Free Vortex Flow. For the free vortex, from equation (5.20), we have vx r= Constant = say ¢ Substituting the value of v in equation (5.23), we get ap = pdr ~ pg de = px dr~pgde=px 5 dr~pg dz Pxr Consider two points 1 and 2 in the fluid having radius r, and r, from the central axis respectively as shown in Fig. 5.27. The heights of the points from bottom of the vessel is z, and z. Integrating the above equation for the points 1 and 2, we get fio or PoP far fog [ Pearmpe fae [Ef mete 3-vi]-psle-a] v? v3] ps [22-2] (5.21) Equation (5.27) is Bernoulli's equation, Hence in case of free vortex flow, Bernoulli's equation is applicable. Fig. 5.27 [210 Fluid Mechanics ] Problem 5.32 In a free cylindrical vortex flow, at a point in the fluid at a radius of 200 mm and at a height of 100 mm, the velocity and pressures are 10 m/s and 117.72 kN/m? absolute. Find the pressure at a radius of 400 mm and at a height of 200 mm. The fluid is air having density equal to 1.24 kg/m’. Solution. At Point 1 : Given Radius, = 200 mm = 0.20 m Height, z= 100 mm = 0.10 m Velocity, 0 mis Pressure, 17.72. kNN/m? = 117.72 x 10° Nim? At Point 2: 400 mm = 0.4 m 2 = 200 mm = 0.2 m Py = pressure at point 2 p= 1.24 kg/m? For the free vortex from equation (5.20), we have vx r= constant oF 7 10x02 ofa mis But p= 1.24 kg/m’ Uu772x10" 10g my aot 124x981" 2x981 pe 2x981 Py MII, 10? yy) 5? _ 9g pe 124x981 2x98” 2x93 9677.4 + 5.096 + 0.1 ~ 1.274 ~ 0.2 = 9676.22 1676.22 x py = 9676.22 x 1.24 x 9.81 17705 N/m? = 117.705 x 10° Nim? 17.705 kN/m? (abs.) = 117.705 kN/m?, Ans. (B) IDEAL FLOW (POTENTIAL FLOW) > 5.11 INTRODUCTION [Wal fluid is a fluid which is incompressible and inviscid. Incompressible fluid is a fluid for which density (p)remains constant. Inviscid fluid is a fluid for which viscosity (1) is zero. Hence a fluid for which density is constant and viscosity is zero, is known as an ideal fluid. ‘The shearstons is given by, t= 1 SE, Hence for idea id the shear ston wll e210 a8 = 0 for ideal fluid. Also the shear force (which is equal to shear stress multiplied by area) will be zero in [ Kinematics of Flow and Ideal Flow 211 | case of ideal or potential flow. The ideal fluids will be moving with uniform velocity. All the fluid particles will be moving with the same velocity. ‘The concept of ideal fluid simplifies the typical mathematical analysis. Fluids such as water and air have low viscosity. Also when the speed of air is appreciably lower than that of sound in it, the compressibility is so low that air is assumed to be incompressible, Hence under certain conditions, certain real fluids such as water and air may be treated like ideal fluids. > 5.12 IMPORTANT CASES OF POTENTIAL FLOW ‘The following are the important cases of potential flow : (i) Uniform flow, (ii) Source flow, (iii) Sink flow, (iv) Free-vortex flow, (v) Superimposed flow. > 5.13 UNIFORM FLOW In a uniform flow, the velocity remains constant. All the fluid particles are moving with the same velocity. The uniform flow may be : (i) Parallel to x-axis Gi) Parallel to y-axis. 5.13.1. Uniform Flow Parallel to x-Axis. Fig. 5.27 (a) shows the uniform flow parallel to a-axis. In a uniform flow, the velocity remains constant. All the fluid particles are moving with the same velocity. Fig. 5.27 (a) Le U = Velocity which is uniform or constant along x-axis uw and y = Components of uniform velocity U along x and y-axis. For the uniform flow, parallel t0 x-axis, the velocity components u and v are given as u=Uandy=0 (5.28) But the velocity uw in terms of stream function is given by, ay and in terms of velocity potential the velocity u is given by, ay _ a ua VL 5.29) a (5.29) 5 av _% Similarly, it can be shown that y = ~ 2¥ = 2% 5.294 ae te oe ay oe) But w= U from equation (5.28). Substituting u = U in equation (5.29), we have [212 Fluid Mechanics ] Le 5.3 u (5.30) & sand also u = 2 or = and atso u = First part gives dy = U dy whereas second part gives d@ = U dx. Integration of these parts gives as We Uy+C, and = Uxt Cy where C, and Cy are constant of integration, Now let us plot the stream lines and potential lines for uniform flow parallel to x-axis. Plotting of Stream lines. For stream lines, the equation is waUxy+G Let y= 0, where y = 0. Substituting these values in the above equation, we get 0=Ux04C, or C,=0 Hence the equation of stream lines becomes as y weU.y (5.31) ‘The stream lines are straight lines parallel to x-axis and at a distance y from the x-axis as shown in Fig. 5.28. In equation (5.31), U. y represents the volume flow rate (i. m/s) be- tween x-axis and that stream line at a distance y. Note. The thickness of the fluid stream perpendicular to the plane is assumed to be unity. Then y x I or y represents the area of flow. And Uy represents the produet of velocity and area. Hence U. y represents the volume flow rate. Stream lines ° Plotting of potential lines. For potential lines, the equation is O=U.x+C, Let @=0, where x= 0. Substituting these values in the above equation, we get C, Hence equation of potential lines becomes as oe Ux ‘The above equation shows that potential lines are straight lines parallel to y-axis and at a distance of x from y-axis as shown in Fig. 5.29. Fig. 5.30 shows the plot of stream lines and potential lines for uniform flow parallel to x-axis. The stream lines and potential lines intersect each other at right angles. be y cle Potental ines esis Potanal sys ines J rts fe 4 tC ers * We vs chee ease wo Fig. 5.29 Fig. 5.30 [ Kinematics of Flow and Ideal Flow 213] 5.13.2 Uniform Potential Flow Parallel to y-Axis. Fig. 5.31 shows the uniform potential flow parallel to y-axis in which U is the uniform velocity along y-axis. Fig. 531 The velocity components 1, v along x-axis and yraxs are given by u=0andv=U (533) ‘These velocity components in tems of steam function (y) and velocity potential function (9) are piven as av _ 2% = A534 dye ae ay _ a penta (5.35) - ee 635) But from equation (5.33), v = U. Substituting v = U in equation (5.35), we get oy _ 4% oy 29 ya oe ya and aso = % a ay By Firs part gives dy = — U ce whereas second part gives p= U dy Integration ofthese pars gives as, W=-U.x+C, and 0=U.y+Cy (5.36) where C, and C, are constant of integration, Let us now plot the stream lines and potential lines. Plotting of Stream lines. For stream lines, the equation is y= U.x + C, Let w= 0, where x= 0. Then C, = 0. Hence the equation of stream lines becomes as y = ~ U.x (3.37) ‘The above equation shows that stream lines are straight lines parallel to y-axis and at adistance of x from. the y-axis as shown in Fig. 5.32. The ve sign shows that the str:am lines are in the downward direction. stream lines I 5 +7 Pojentatinos Ys al ‘ [214 Fluid Mechanics ] Plotting of Potential lines. For potential lines, the equation is 6 = U.y + C, Let = 0, where y = 0. Then C; = 0. Hence equation of potential lines becomes as 6 = U.y (5.38) ‘The above equation shows that potential lines are straight lines parallel to x-axis and at a distance of 1y from the x-axis as shown in Fig. 5.32. > 5.14 SOURCE FLOW ‘The source flow is the flow coming from a point (source) and, moving out radially in all directions of a plane at uniform rate. Fig. 5.33 shows a source flow in which the point 0 is the source from which the fluid moves radially outward. The strength of a ol source is defined as the volume flow rate per unit depth, The unit 3 ‘ of strength of source is m/s. It is represented by 4. oer Let u, = radial velocity of flow at a radius r from the source O “(sy volume flow rate per unit depth dius ‘The radial velocity u, at any radius r is given by, Fig. 5.33 Source low (Flow atway (5.39) from source) ‘The above equation shows that with the increase of r, the radial velocity decreases. And at a large distance away from the source, the velocity will be approximately equal to zero, The flow is in radial direction, hence the tangential velocity ug = 0. Let us now find the equation of stream function and velocity potential function for the source flow. As in this case, up = 0, the equation of stream function and velocity potential function will be obtained from u, Equation of Stream Function By definition, the radial velocity and tangential velocity components in terms of stream function are given by [See equation (5.124)] But [See equation (5.39)] p24 or A w= hal nr? Qn a Integrating the above equation wat. 8, we gt w= dh x0 6 Cy, whee C, i constant of isp Let y=0, when 0 =0, then C, =0. Hence the equation of stream function becomes as Kinematics of Flow and Ideal Flow 215 4 at 4 vege (5.40) In the above equation, q is constant. ‘The above equation shows that stream function is a function of 8. For a given value of , the stream function will be constant, And this will be a radial line. The stream lines ean be plotted by having different values of @. Here 0 is taken in radians. Plotting of stream lines sa When @=0,y=0 Stream ines 0= 45° = = radians, y=. = =4 units ‘are radal 4 m4 ok q m4 vo 0= 90° = radians, y=. =4 units 4 2 me oe 24 a Equation of Potent wai By definition, the radial and tangential components in Fig. 5.34 Stream line for terms of velocity function are given by source flow. [See equation (5.94)] ar But from equation (5.39), u,= nr Equating the two values of u, we get og 4 2 or aye ar ar wr Or Integrating the above equation, we get ja-[fa Potent ines 2nr mere afl 4 or Ap dar fis Lisaconstan tem ante | ae 4 A top,r St Qn Be (S41) In the above equation, q is constant. ‘The above equation shows, that the velocity potential function is ‘a function of r. For a given value of r, the velocity function 6 will be constant. Hence it will be a circle with origin at the source, The velocity potential lines will be circles with origin at the source as shown in Fig. 5.35. Let us now find an expression for the pressure in terms of radius. Fig. 5.35. Potential lines for source [216 Fluid Mechanics ] Pressure istribution in a plane source flow ‘The pressure distribution in a plane source flow can be obtained with the help of Bernoulli's equa- tion. Let us assume that the plane of the flow is horizontal. In that case the datum head will be same for two points of flow. Let p= pressure at a point 1 which is at a radius r from the source at point 1 1, = velocity at point 1 Po = Pressure at point 2, which is at a large distance away from the source. The velocity will be zero at point 2. [Refer to equation (5.39)] Applying Bernoulli's equation, we get Pt Po gy gq (PAB) ps 28 pg Pg or (pp) =-= Bat fom enusion(S.3), 4, = 52 (6) 2) \ ar, In the above equation, p and q are constants. ‘The above equation shows that the pressure is inversely proportional to the square of the radius from the source. (5.42) > 5.15 SINK FLOW The sink flow is the flow in which fluid moves radially inwards towards a point where it disappears at a constant rate. This flow is just opposite t0 the source flow. Fig. 5.36 shows 4 sink flow in which the fluid moves radially inwards towards Point O, where it disappears at a constant rate. The pattern of stream lines and equipotential lines of a sink flow is the same as that of a source flow. All the equations derived for a source Sink ra flow shall hold to good for sink flow also except that in sink flow equations, q is to be replaced by (~ q). Problem 5.33 Plot the stream lines for a uniform flow of (5 m/s parallel to the positive direction of the x-axis and Fig.5.36 Sink flow (ii) 10 ms parallel to the positive direction of the y-axis. (Flow toward centre) Solution. (i) The stream function for a uniform flow parallel to the positive direction of the avaxis is given by equation (5.31) as w=Uxy The above equation shows that stream lines are straight lines parallel to the x-axis at a distance y from the x-axis. Here U = 5 mis and hence above equation becomes as Kinematics of Flow and Ideal Flow 217 w= sy For y=0, stream function y= 0 For y= 02, stream function y= 5 %0.2=1 unit For y= 044, stream function y= 5 x0.4 = 2 unit The other values of stream function can be obtained by substituting the different values of y. The stream lines are horizontal as shown in Fig. 5.36 (a). ¥ 0g; + y=08 06 ee y=08 04, + y=0a 02} + y=02 ° yo Fig. 8.36 (a) (Gi The stream function fora uniform flow parallel tothe positive direction ofthe y-axis is given by equation (5.37) as, wo-Uxx The above equation shows that stream lines are straight lines parallel to the y-axis at a distance x from the y-axis. Here U = 10 mvs and hence the above equation becomes as ye-l0xx ‘The negative sign shows that the stream lines are in the downward direction. For x=0, — the stream function y= 0 For x= 0.1, the stream function y =~ 10 x 0.1 =~ 1.0 unit For x= 0.2, the stream function y =~ 10x 0.2 =~ 2.0 unit For x= 0.3, the stream function y =~ 10 x 0.3 =~ 3.0 unit ‘The other values of stream function can be obtained by substituting the different values of x. The stream lines are vertical as shown in Fig. 5.36 (b).. 0 01 02 03 Fig. 5.36 (b) [218 Fluid Mechanics ] Problem 5.34 Determine the velocity of flow at radii of 0.2 m, 0.4 m and 0.8 m, when the water is flowing radially outward in a horizontal plane from a source at a strength of 12 m/s. Solution. Given : Strength of source, q = 12 m’is ‘The radial velocity wat any radius r is given by equation (5.39) as, sz nr When r= 0.2 m, Z 9.55 m/s. Ans. In x02 When r= 04 m, = 4.77 mis. Ans. Inxoa When r= 0.8 m, a 2.38 m/s. Ans. 2m x08 Problem §.35 Two discs are placed in a horizontal plane, one over the other. The water enters at the centre of the lower disc and flows radially outward from a source of strength 0.628 m*/s. The pressure, at a radius 50 mm, is 200 kN/m*, Find : (i) pressure in kN/m’ at a radius of 500 mm and (ii) stream function at angles of 30° and 60° if y= 0 at 8 = 0°. Solution. Given : Source strength, q= 0.628 m/s Pressure at radius 50 mm, p, = 200 kN/m’ (@ Pressure at a radius $00 mm Let p= pressure at radius 500 mm (u,), = velocity at radius 50 mm (u,), = velocity at radius 500 mm The radial velocity at any radius r is given by equation (5.39) as 100 x 10° Nim? When r= 50mm =0.05m, (u,), = 288 = 1.998 mis = 2 mis 2m x005 When r= 500mm =0.5m, (u,),= 28 = 0.2 mis 2x05 Applying Bernoulli's equation at radius 0.05 m and at radius 0.5 m, or Kinematics of Flow and Ideal Flow 219 or or +002 or = 202 - 0.02 = 201.98 1000 P:= 201.98 x 1000 Nim? = 201.98 kNim?. Ans. (Gd Stream functions at @= 30° and @= 60° For the source flow, the equation of stream function is given by equation (5.40) as w= £0, where 0 is in radians 2n 0.628 30x 30xn ) When @ = 30°, oe x radians Ve oe 180 80 = 0.0523 m?is. Ans. 0.628 60m 2 Se gp = 01046 ms, Ans. When @ = 60°, v > 5.16 FREE-VORTEX FLOW Free-vortex flow is a circulatory flow of a fluid such that its stream lines are concentric circles. For a free-vortex flow, ug x r = constant (say C) Also, circulation around a stream line of an irrotation vortex is T= 2nr x ug=2nxC where ug = tangential velocity at any radius r from the centre, et O° ar ‘The circulation T’is taken positive if the free vortex is anticlockwise. For a free-vortex flow, the velocity components are 1rXup= ©) Equation of Stream Function By definition, the stream function is given by ay ar In case of free-vortex flow, the radial velocity (1, is zero. Hence equation of stream function will be obtained from tangential velocity, ug, The value of uy is given by r nr Equating the two values of ug, we get Ug = and [See equation (5.124)] up [220 Fluid Mechanics ] a or 2nr 2nr Integrating the above equation, we get or (+ Eisacamanr em) 65) B ‘The above equation shows that stream function is a function of rn Foragionslocof ects icon constant Hone the stream lines are concentric circles as shown in Fig. 5.37. ‘Stream yea af poco foto By dees th poi me tucton Ben nae EB ant 238 Sc quien 3:94) Beet aloe ana eee oterecients: poeatl ete? 7 Qnr ie 5 function wil be obtained from up. TEquating the two value of up We get 13 _ 7 r r +O “ Ie On Integrating the above equation, we get r r = fla or o=Efa Jo= Sas 2 ‘The above equation shows that velocity potential function isa function of @. For a given value of @, potential function is a constant. Hence equipotential lines are radial as shown in Fig. 5.38. Velocty potential tines 6.44) Fig. 5.38 Potential lines are radial. Kinematics of Flow and Ideal Flow 221 > 5.17 SUPER-IMPOSED FLOW ‘The flow patterns due to uniform flow, a source flow, a sink flow and a free vortex flow can be super-imposed in any linear combination to get a resultant flow which closely resembles the flow around bodies. The resultant flow will still be potential and ideal. The following are the important super-imposed flow : (® Source and sink pair (ii) Doublet (special case of source and sink combination) (ii), A plane source in a uniform flow (flow past a half body) iv) A source and sink pair in a uniform flow (©) A doublet in a uniform flow. 5.17.1 Source and Sink Pair. Fig. 5.39 shows a source and a sink of strength q and (~ q) placed at A and B respectively at equal distance from the point O on the x-axis. Thus the source and sink are placed symmetrically on the x-axis. The source of strength q is placed at A and sink of strength (— q) is placed at B. The combination of the source and the sink would result in a flownet where stream lines will be circular ares starting from point A and ending at point B as shown in Fig. 540. o ‘ 3 7 (; Miss ca i. _. es fete a os = - Fig. 5A Stream lines fr sourcesink pair. Equation of stream function and potential function Let P be any point in the resultant flownet of source and sink as shown in Fig. 5.41 [222 Fluid Mechanics Fig. 5.41 Let, @ = Cylindrical co-ordinates of point P with respect to origin O x.) = Corresponding co-ordinates of point P 171.8; = Position of point P with respect to source placed at A ‘ry, 8, = Position of point P with respect to sink placed at B = Angle subtended at P by the join of source and sink i.e., angle APB. Let us find the equation for the resultant stream function and velocity potential function. The ‘equation for stream function due to source is given by equation (5.40) as y, = 4-8 whereas due to sink itis given by these two stream function. = 24a [a= 6,—6).Iniriangle ABP, 0, + 0 + (180° 8) an =180° 3. @=0,- 6) a 5 5.45 £ (5.45) ‘The equation for potential function due to source is given by equation (5.41) as 6, pal log -r, and $e tog.ry. The equation for resultant potential function (@) will be the sum of these two potential function. OR hitb due to sink it is given as 6, 4 =a Lvog.r + ($4) 02.72 an ee ) me Kinematics of Flow and Ideal Flow 223 = E toss, lorerd= os, (8) (5.46) 2n b ‘To prove that resultant stream lines will be circular are passing through source and sink ‘The resultant stream function is given by equation (5.45) as 4.0 2n v 4 <4 @,-6) = 0, - 8, Ze OO) a ) For a given stream line y = constant. In the above equation the term rs is also constant. This ‘means that (Q) ~ 64) of angle « will also be constant for various positions of P in the plane. To satisfy this, the locus of P must be a circle with AB as chord, having its centre on y-axis, as, shown in Fig. 5.40. ‘Consider the equation (5.45) again as, (a= 6-0) or Taking tangent to both sides, we get tan (@, ~ 03) = tan (=) “ 4 Bu tan 8, = (S46) rea Substituting the values of tan 8, and tan @, in equation (), y (ea) y Gre =a) 1+ oe v(e~a)= v(eta) or or ) Fig. 5.41 (a) [224 Fluid Mechanics ] or Poa tay cot (24) a0 a a 24 y+ Day cot =) - q or ay tay ct (284) «cor (2 a a e+|reeca 224) [2cose(224)/ san The above is the equation ofa cicle* with centre on y-axis ata distance of + a cot (=) from q the origin. The ais Fie ce wil be a soe (28), q Similarly, it can be shown that the potential lines for the source-sink pair will be eccentric non- intersecting circles with their centres on the x-axis as shown in Fig. 5.41 (6). ¥ Potential ins Fig. 541 6) Potential lines for source sink pair (Potential lines are eccentric noncintersecting circles with their centres on x-axis) "The equation x7 + y? = a? is the equation of a circle with centre at origin and of radius ‘a Kinematics of Flow and Ideal Flow 225 Problem 5.36 A source and a sink of strength 4 m°/s and 8 m*/s are located at (~ 1, 0) and (1, 0) respectively. Determine the velocity and stream function at a point P (1, 1) which is lying on the ‘lownet of the resultant stream line. Solution, Given : Source strength, q=4 mis Sink strength, q= 8 ms Distance of the source and sink from origin, a = 1 unit ‘The position of the source, sink and point P in the flow field is shown in Fig. 5.42. From Fig. 5.42, itis clear that angle 0, will be 90° and angle 8, can be calculated from right angled triangle ABP. ‘The equation for stream function due to source is given by equation (5.40) as y, = 1 x = 4 x8, rd whereas due to sink it is given by yp ‘The resultant stream function w is given as voy + ve eli) Let us find the values of 8, and 8, in radians. From the geometry, itis clear that the traingle ABP is a right angled tiangle with angle 0, = 90° = 20. xm = radians 180 *"" 2 BP LA ‘Also an, = FES or Substituting these values in equation (i), a. a = 4 x 0463 - 2x v Qn Qn 2 [226 Fluid Mechanics ] m’Js, q> = 8 m’/s) k 8 == xo.463-& an 048 3E* 9 = 0.294 ~ 2 106 m/s. Ans. To find the velocity at the point P, let us first find the stream function in terms of x and y co- ordinates. The stream function in terms of ®, and @, is given by equation (i) above as = 1X _ bx On Oe The values of @, and @, in terms of x, y and a are given by equation (5.464) as tan 8, or a The velocity component 2m (x= a) +y ‘At the point P(1, 1), the component 1 is obtained by subs equation. The value of a is also equal to one. the above ituting x = 1 and y (-1) Kinematics of Flow and Ideal Flow 227 Now a (xtay 2m (x+a) ty oh» Qn (xtay + At the point P(L, 1). ye yt ___ ——— Qn (i+ly +P 2e (=I) +P Myla) 2n5 201 ai 4.1 8 2 fxt-Bet xl-F 20072-1272 =~ 1.14 Pox SEA go dg = 01272 ~ 1272 = ~ 1.145 mis ‘The resultant velocity, V= fu? +v* = 0.2544? +(-1145)" = 1.174 mis. Ans. Problem 8.37 For the above problem, determine the pressure at P(1, 1) ifthe pressure at infinity is zero and density of fluid is 1000 kg/m’ Solution, Pressure at infinity, pp Density of fluid, = 1000 kg/m® The velocity* of fluid at infinity will be zero. If Vp = velocity a infinity, then Vo = 0. The resultant velocity of fluid at PCL, 1) = 1.174 ins (calculated above) or Ve 1174 ms. Let p = pressure at P(1, 1) Applying Bernoulli's theorem at point at infinity and at point P, we get Fo,Mo PV pg 2g pe or o+o-2 or V= 1.174 mis) * From equation (5.39), the velocity ata distance ‘from source or sink is given by u,= 4. At infinity, ris very very large hence velocity is zero. [228 Fluid Mechanics __ 1174? 1000 ~ 2 5.17.2 Doublet. It isa special case of a source and sink pair (both of them are of equal strength) ‘when the two approach each other in such a way that the distance 2a between them approaches zero and the product 2a . q remains constant. This product 2a . q is known as doublet strength and is denoted by 1. Doublet strength, w= 2a.g (5.48) Let q and (~ q) may be the strength of the source and the sink respectively as shown in Fig. 5.43. Let 2a be the distance betweem them and P be any point in the combined field of source and sink. or 689.14 Nim’, Ans. Fig. 543, Let @ is the angle made by P at A whereas (@ + 66) is the angle at B. Now the stream function at P, 4 fe (3.49) From B, draw BC Jon AP. Let AC = 8r, CP = rand AP = r+ Br. Also angle BPC = 88. The angle {80 is very small. The distance BC can be taken equal to r x 88. In triangle ABC, angle BCA = 90° and hence distance BC is also equal to 2a . sin @, Equating the two values of BC, we get x 80 = 2a. sin @ 2a.sin® 0 Substituting the value of 69 in equation (5.49), we get _ 4, 2asin® nr A on In Fig. 5.43, when 2a — 0, the angle 88 subtended by point P with A and B becomes very small. Also dr — 0 and AP becomes equal to r. Then v [ie 2a. q = from equation (5.48)] ..(5.50) Subst ting the value of sin @ in equation (5.50), we get Kinematics of Flow and Ideal Flow 229 hy Pavey) referer) 6 » (5.504) or The above equation can be written as Fever gie(h) (a) <0 [sine an sabre (2) any * (any, any or e+(+4) (4) (5.51) H The above is the equation of a circle with centre {0,=4—) and radius 4. The centre of the au es ce (0 i) aati Pte ee Circle lies on y-axis ata distance of —— from x-axis. As the radius ofthe circle is also equal to —#—, any 4m hence the citcle will be tangent to the x-axis. Hence stream lines of the doublet will be the family of circles tangent to the x-axis as shown in Fig. 5.44, Stream ines are circles ‘tangent to x-axis with conte on y-axis ‘Stream ines y toes Fig. 5.44 Stream lines for a doublet. Potential function at P Refer to Fig. 5.43. The potential function at P is given by o= Log r+8n+ (- 4 og.r [Refer to equation (5.41)] on 2x [230 Fluid Mechanics = Litog, (r+ 6)- Log r= tog, (78) = to ( a = Ltos.(r+ an £ toe. to, (: =e. 1b : o£ |&+(%) xb... =£.% _[neSEisa sma qumiy.ten( 5) tems ns] Dot in gS. om wine 4, we et = cs ‘. br = 2a cos @ Substituting the value of Br, we get a Hy 24.0088 Qn 038 x nr In Fig. 5.43, when 2a — 0, the angle 68 becomes very small. Also r — 0 and AP becomes equal to r. Then [2a x q = from equation (i) ...5.52) on = 42 Also AP* = AD? + PD? ot ? = 1°+ y? Substituting the value of cos @ in equation (5.52), we get vey o 0 ‘The above equation can be writen as Fl . ets (4) (8 [ase and svat (5) ] 2n (amp) ~ (ano, ‘and, a ro) aye (LY 5.53) (a) +¥=(5) (3.53) ‘The above is the equation of a circle with centre (40) and radius {The centre of the ano ano u circle lies on x-axis at a distance of 4 from y-axis. As the radius of the circle is equal to the distance of the centre of the circle from the y-axis, hence the circle will be tangent to the y-axis. * Expansion of log, (1 +x)=x+ = ‘pa gL ta) xt Kinematics of Flow and Ideal Flow 231 Hence the potential lines of a doublet will be a family of circles tangent to the y-axis with their centres oon the x-axis as shown in Fig. 5.45, Porn! nes oF ines ae circles vith centre on axis buttangont to y-axis Potential ines Fig. 5.45 Potential lines for a doublet. Problem 5.38 A point P(0.5, 1) is situated in the flow field of a doublet of strength 5 m/s. Calculate the velocity at this point and also the value of the stream function. Solution. Given : Point P(0.5, 1). This means x = 0.5 and ‘Strength of doublet, B= Sms (®) Velocity at point P ‘The velocity at the given point can be obtained if we know the stream function (y). But stream function is given by equation (5.504) a8 H. y 2 (ey) ‘The velocity components w and v are obtained from the stream function as 10 we- and [232 Fluid Mechanics ] “keel eleoy ‘Substituting the values of p= 5 m7/s, x = 0.5 and y = 1.0, wet), (os? +17) Resultant velocity, V= yu? +? = y(-0382)' +(-0.509)" = 0.636 mvs. Ans. (i Value of stream function at point P , we get the velocity components as and 2n Solution in polar co: The above question can also be done in r, 0 (ie, polar) co-ordinates. The stream function in r, 8 co-ordinates is given by equation (5.50) as eli) [+ isa contanttxm and also sconstant v.40] 2 1 =r cos 0 ii) zal Eee and. = arl on 8 isa constant wor.t.r (iti) Kinematics of Flow and Ideal Flow 233 Now rex? ty? = 0s? +P =Vi25 : sin @= 2=—1_ = 0,894 and cos 6 =~ ris r Substituting the values ofr, sin @ and eos 6 in above equations (i, (i) and (ii), we get ht sin@_ 5 0894 - x TEE = - 0.636 mis. Ans. Woon rom Vis x05 @=- x1 x 0,447 = - 0.2845 mis 2n ” (125) and. Resultant velocity, 0569)? = 0.636 m/s. Ans. 5.17.3 A Plane Source in a Uniform Flow (Flow Past a Half-Body). Fig. 5.46 (a) shows uniform flow of velocity U and Fig. 5.46 (b) shows a source flow of strength q. When this uniform flow is flowing over the source flow, a resultant flow will be obtained as shown in Fig. 5.46. This resultant flow is also known as the flow past a half-body. Let the source is placed on the origin O. Consider a point P(x, y) lying in the resultant flow field with polar co-ordinates and @ as shown in Fig. 5.46. y @) (2) Unitorm fow (©) Source fow Hatfbody Point Px. y) Fig. 5.46 Flow pattern resulting from the combination of a uniform flow and a source. [234 Fluid Mechanics ] ‘The stream function (y) and potential function (6) for the resultant flow are obtained as given below : \y = Stream function due to uniform flow + stream funetion due to source su.y+ Lo (5.54) 2n =U. rsind+ £0 y= sin @) (5.54) 2x and = Velocity potential function due to uniform flow + Velocity potential function due to source 4 5s. =U.xt A ogy =U .rcos 0+ 4 tog,r 5.548) 2 Be 2m ee (S.54B) ‘The following are the important points for the resultant flow pattern : (® Stagnation point. On the left side of the source, at the point lying on the x-axis, the velocity of uniform flow and that due to source are equal and opposite to each other. Hence the net velocity of the combined flow field is zero. This point is known as stagnation point and is denoted by S. The polar co- ordinates of the stagnation point S are rs and , where rs is radial distance of point S from 0. ‘The net velocity (or resultant velocity) is zero at the stagnation point S. 1 a1? fore 2°) [ =U.rsino+ £6] Para (Urano 2n = {[urcosoe £]=v.cos 0+ r 2 ar ‘At the stagnation point, ® = m radians (180°) and r ‘sand net velocity is zero. This means u, = 0 and vp = 0. Substituting these values in the above equation, we get 0=U.. cos 180° + 4 [> u,=0, 8 = 180° and r= re} 2s =-U+— on er 4 or neo (5.55) Inv cane From the above equation it is clear that position of stagnation point depends upon the free stream velocity U and source strength q. At the stagnation point, the value of stream function is obtained from equation (5.544) as 4 =U.rsinos 1.0 V Qn For the stagnation point, the above equation becomes as u.r,sin 190° + 1 x8 2n [At stagnation point, @ = x radians = 180° and r = rs] so+te4 2° 2 ‘The above relation gives the equation of stream line passing through stagnation point. We know that no fluid mass crosses a stream line. Hence a stream line is a virtual solid surface. (5.56) Kinematics of Flow and Ideal Flow 235 (i Shape of resultant flow. At the stagnation point S, the net velocity is zero. The fluid particles that issue from the source cannot proceed further to the left of stagnation point. They are carried along the contour BSB’ that separates the source flow from uniform flow. The curve BSB’ can be regarded as the solid boundary of a round nosed body such as a bridge pier around which the uniform flow is forced to pass. The contour BSB’ is called the half body, because it has only the leading point, it trails to infinity at down stream end. ‘The value of stream function of the stream line passing through stagnation point S and pas 4 the solid boundary (i.e., curve BSB’) is Ws = ‘Thus the composite flow consists of : (1) flow over a plane half-body (i.e, flow over curve BSB’) outside y (2) source flow within the plane half-body. The plane half-body is deseribed by the dividing steam line, y = 4. But the stream function at any point in the combined flow field is given by equation (5.54) as, 4 =U.y+ Le weve oe If we take y= 4 in the above equation, we will get the equation of the dividing stream line. 2 ++ Equation of the dividing stream line (i.e., equation of curve BSB’) will be or (3.57) From te sbove equation, the main dimcasons of he plate aly maybe obaned. rom hs equation, iis ea thay maximum, hen 8 =O Hence A19=0, IRAN ag = the max ode the ordinate above the origin the leading point of the half-body =, y = the ordinate below the origin. ‘The main dimensions are shown in Fig. 5.47. (iii) Resultant velocity at any point ‘The velocity components at any point in the flow field are given by 1-1 dl y.rsino+ Lo] “7 ae de Qn [236 Fluid Mechanics ] [verso =U.c0s 0+ 2 . 2nr Ordinate above the origin Plane halt-body Fig.5.47 ‘The above equation gives the radial velocity at any point in the flow field. This radial velocity is due {© uniform flow and due to souree. Due to source the radial velocity is =4. Hence the velocity due nr fance from the to source diminishes with increase in radial distance from the source. At large di source the contribution of source is negligible and hence free stream uniform flow is not influenced by the presence of source. W.-2ferinee Lo] Oe Oe 2 [U. sin 8 + 0] =-U sin @ Resultant velocity, V= i tug (iv) Location of stagnation point At the stagnation point, the velocity components are zero. Hence equating the ri velocity components to zero, we get 4,20 or Ucos0+—L =0 or Ucos nr But rcos or reos @=- 4 2nu 4q 2a 0 or sin@ rsin 8 y ‘as U cannot be zero Kinematics of Flow and Ideal Flow _237 (0) Pressure at any point in flow field Let pp pressure at infinity where velocity is U ’P= pressure at any point P in the flow field, where velocity is V Now applying the Bemouli's equation at a poin at infinity and ata point P inthe flow field, we get rove gy Vp po p- Pe pe 2e pe 2e 2g 28 PR Pk pe The pressure co-efficient is defined as (5.58) Problem 5.39 A uniform flow with a velocity of 3 m/s is flowing over a plane source of strength 30 m*/s. The uniform flow and source flow are in the same plane. A point P is situated in the flow ‘field. The distance of the point P from the source is 0.5 m and itis at an angle of 30° to the uniform ‘low. Determine : (i) stream function at point P, (ii) resultant velocity of flow at P and (iii) location of stagnation point from the source. Solution, Given : Uniform velocity, U = 3 m/s ; source strength, q = 30 m’/s ; co-ordinates of point P are r= 0.5 m and @ = 30°. (0 Stream function at point P The stream function at any point in the combined flow field is given by equation (5.544) 4 =U. rsinos Le V Qn a 159m = Fig. 548, ‘Stream function at point P, ve anossin a 2x( 23) ain [238 Fluid Mechanics ] = 015425 = .25 mis. Ans. (ii Resultant velocity at P The velocity components anywhere in the flow are given by Low 14 q W148 ly sins Lo 708738 “OR = [u.rcose+]=u.cose + 2. 2nr 30 2exO5 2.14 = 3 xcos 30° + » AUP, r= 05,0 = 30°, q = 30) = 2.598 49.55 Tunes Lol Bean jUrsnos ol Usin®+0=-Usin@ =~ 3xsin 30° =- 1.5 = 124? +(-15)° = 12.24 mvs. Ans. (iii) Location of stagnation point The horizontal distance of the stagnation point S from the source is given by equation (5.55) as aoe aU 2Rxd The stagnation point will be at a distance of 1.59 m to the left side of the source on the x-axis Problem 5.40 A uniform flow with a velocity of 20 m/s is flowing over a source of strength 10 m’/s. The uniform flow and source flow are in the same plane. Obtain the equation of the dividing stream line and skeich the flow pattern Solution. Given : Uniform velocity, U = 20 m/s ; Source strength, q = 10 m’/s (i) Equation of the dividing stream line ‘The stream function at any point in the combined flow field is given by equation (5.544) =U.rsino+ v=U.rsind+ 1 0 and. ug= Resultant velocity, V 1.59 m. Ans. ts =2xrsino+ 29 2x U = 20 mis and q = 10 mss) ‘The value of the stream function for the dividing stream line is y= 4. Hence substituting y = the above equation, we get the equation of the dividing stream line. 4 2 or sin d+ 12.9 2x 10 or = 200 sin 0+ 5 Cr q= 10) Kinematics of Flow and Ideal Flow 239 or 5=20rsin 0+ 12 6= 207+ 26 G rsin@=y) 2n an 10 W0y=5-—O Po Oe 6 6 =o25- 2 Ai or s 2 “a ‘The above relation gives the equation of the dividing stream line. From the above equation, for different values of 8 the value of y is obtained as Value of © | Value of v from (i Remarks 0 025 m ‘Max. half width of body z 0.125 m ‘The +ve ordinate above the origin ® 0 ‘The leading point * = 0.125 m ‘The ~ve ordinate below the origin 2m. = 025 m ‘The max. ~ve ondinate (i) Sketch of fiow pattern For sketching the flow pattern, let us first find the location of the stagnation point, The horizontal distance of the stagnation point S from the source is given by the equation, q __ 0 2nU 2nx20 Hence the stagnation point lies on the x-axis at a distance of 0.0795 m or 79.5 mm from the source towards left of the source. The flow pattern is shown in Fig. 5.49. _————————— ee 0.0795 m 735 mm Fig. 5.49 Problem 5.41 A uniform flow with a velocity of 2 m/s is flowing over a source placed at the origin. The stagnation point occurs at (~ 0.398, 0). Determine : (i) Strength of the source, (ii) Maximum width of Rankine half-body and (iti) Other principal dimensions of the Rankine half-body. Solution, Given : Uniform velocity, U=2mis [240 Fluid Mechanics ] Co-ordinates of stagnation point = (~ 0.398, 0) ‘This means r, = 0.398 and stagnation point lies on x-axis at a distance of 0.398 m towards left of, origin, The source is placed at origin. (O Strength of the source Let q = strength of the source In equation (5.70), r= R} and |x @=-2U sin® (5.73) —ve sign shows the clockwise direction of tangential velocity at that point. The value of ug is maximum, when 8 = 90° and 270°, ‘At 8 = 0° or 180°, the value of up = 0. Hence on the surface of the cylinder, the resultant velocity is zero, when @ = 0° or 180°. These two points on the surface of cylinder [i-e., at © = 0° and 180°] where resultant velocity is zero, are known as stagnation points. They are denoted by 5, and Sy. Stagnation point S, corresponds to @ = 180° and S; corresponds to = 0°. Pressure distribution on the surface of the cylinder Let pg = pressure at a point in the uniform flow far away from the cylinder and towards the left of the cylinder {ie., approaching uniform flow] U = velocity of uniform flow at that point P= pressure at a point on the surface of the cylinder V = resultant velocity at that point on the surface of the cylinder. This velocity is equal to lug as u, is zero on the surface of the cylinder. Kinematics of Flow and Ideal Flow 249 x V= y= 2U sin @ Applying Bemouli's equation atthe above two points, ve 24 [-2Usin ef! or [Ve ug=~2U sin 0] Lip 2 or Sut asin?) 5% ) or foe 3 Pu’ But £—Pe js a dimensionless term and is known as dimensionless pressure co-efficient and is Tou? 3° denoted by C,. fF 1 -4sin?@ Toy? 30" Value of pressure co-efficient for different values of 6 Value of 8 Value of ° 1-4sin?@ =1-0=1 30° 14 sin® 30° 90° 14 sin? 90° = 130° 1-4 sin® 150° 180° 1-4 sin? 180° = 1-0=1 At ©= 0 and 180°, there are stagnation points) and 5; respectively ‘At @= 30° and 150°, the pressure co-efficient is zero. At @= 90°, the pressure co-efficient is ~ 3 (ke. least pressure) ‘The variation of pressure co-efficient along the surface of the cylinder for different values of @ are shown in Fig. 5.54, [250 Fluid Mechanics ‘The positive pressure is acting normal to the surface and towards the surface of the cylinder whereas the negative pressure is acting normal to the surface and away from the surface of the cylinder as shown in Fig. 5.55. pressure pressure Fig. 5.55 Problem 5.43 A uniform flow of 12 més is flowing over a doublet of strength 18 m*/s. The doublet is in the line of the uniform flow. Determine : (i) shape of the Rankine oval (ii) radius of the Rankine circle (iti) value of stream line function at Rankine circle (iv) resultant velocity at a point on the Rankine circle at an angle of 30° from x-axis (v) value of maximum velocity on the Rankine circle and location of the point where velocity is max. Solution. Given : U= 12 mis; w= 18 mils (O Shape of the Rankine oval ‘When a uniform flow is flowing over a doublet and doublet and uniform flow are in line, then the shape of the Rankine oval will be a circle of radiu Kinematics of Flow and Ideal Flow 251 (i Radius of the Rankine circle i 8 2nU ~ \2nx12 (ii) Value of stream line function at the Rankine circle ‘The value of stream line function (¥y) at the Rankine circle is zero i.e., w= 0. (iv) Resultant velocity on the surface of the circle, when @ = 30° On the surface of the eylinder, the radial velocity (u,) is zero, The tangential velocity (ug) is given by equation (5.73) as Rer= 2U sin @ =~ 2x 12 x sin 30° = - 12 mvs. Ans, ve sign shows the clockwise direction of tangential velocity at that point Resultant velocity, V= qu tug = yO? +(-12) = 12 mvs. Ans. (0) Maximum velocity and its location ‘The resultant velocity at any point on the surface of the cylinder is equal to wg. But ug is given by, ug =~ 2U sin @ This velocity will be maximum, when @ = 90°. Max. velocity =-2U=-2*12=-24 mis, Ans. Problem 5.44 A uniform flow of 10 més is flowing over a doublet of strength 15 m’/s. The doublet is inthe line of the uniform flow. The polar co-ordinates of a point P in the flow field are 0.9 m and 30°. Find : (i) stream line function and (i) the resultant velocity at the point. Solution. Given : U = 15 mis; r= 0.9 m and Let us frst find the radius (R) of the Rankine circle. This is given by R= BS} 0.488 m Vand” \2mxi0 ‘The polar co-ordinates of the point P are 0.9 m and 30°. Hence r= 0.9 mand @ = 30°. AS the value of r is more than the radius of the Rankine circle, hence point P lies outside the cylinder. (® Value of stream line function at the point P ‘The stream line function for the composite flow at any point is given by equation (5.69) as P(0.9 m, 20°) 252. Fluid Mechanics v-u(-2)ane as } sin 30°C r= 0.9 m, R = 0.488 and @ = 30°) =10 (09- = 10(0.9 - 0.2646) x ; = 3177 mils, Ans. (ii) Resultant velocity at the point P ‘The radial velocity and tangential velocity at any point in the flow field are given by equations (5.70) and (5.71) respectively. : wnt (i-8) eee +4ve sign shows the radial velocity is outward. R and menu (ie sin fe Sign shows the clockwise direction of tangential velocity. Resultant velocity, era oir? + (647) = 373344086 = 8.89 m/s. Ans. v Kinematics of Flow and Ideal Flow 253 2 % 10, te 2 13. rs 16. 17. ‘Continuity equation in differentiat form, ea = = 0 for three-dimensional flow 0 for two-dimensional flow. a ‘The components of acceleration in x, y and z direction are au, du twee av av ‘The components of velocity in 2, y and z direction in terms of velocity potential (6) are pad 2% snd ay ar ‘Te steam function (y) is defined only for two-dimensional ow. The velocity components ins and 2 ag y= lreetins in temas of team fanetion are u =~ 2% and v= 2Y. . ay ar Angular deformation or shear sin rate is given as strain rate = 2 | 4 2H serena 2-2] Rotational components of a fluid ps oy i Vorticity is two times the value of roution Flow ofa fluid along a curved path is known as vortex flow. Ifthe particles are moving round in curved path with the help of some extemal torque the flow is ealled forced vortex flow. And if no extemal torque is required to rotate the Muid particles, the flow is called ftee-vortex flow. ‘The relation between tangential velocity and radius: for forced vortex, y= Ox, for free vortex, r= constant. ‘The pressre variation along the radial direction for vortex flow along a horizontal plane, 22. ar a ‘and pressure variation in the vertical plane 22 = — pe, a For the forced vortex flow, Z= —— ee 2g 2e le where Z= height of paraboloid formed = angular velocity, 254 Fluid Mechanics EXERCISE (A) THEORETICAL PROBLEMS 4, What are the methods of describing fluid flow ? 2. Explain the terms : (@ Path line, (di) Streak line, iif) Stream line, and (iv) Stream tube, Distinguish between (@ Steady flow and un-steady flow, (i) Uniform and non- (iii) Compressible and incompressible flow, (Gv) Rotational and irrotational flow, (v) Laminar and turbulent flow. 4. Define the following and give one practical example for each 3m flow, (@) Laminar flow, (Gi) Turbulent flow, (iid) Steady flow, and (i) Uniform flow. 5. Define the equation of continuity. Obtain an expression for continuity equation for a three-dimensional flow. (R.G.P.V, S 2002) 16 What do you understand by the terms ; () Total acceleration, (ii) Convective acceleration, and (ii) Local acceleration ? (Delhi University, Dec. 2002) 7. (a) Define the terms = (i) Velocity potential function, and (i) Stream function. (®) What are the conditions for flow to be inrotational ? '8 What do you mean by equipotential line and a line of constant stream function ? 9 (a) Deseribe the use and limitations of the flow nels. (©) Under what conditions can one draw flow net ? Define the terms : (Vortex ow, i) Forced vortex flow, and (ii) Free vortex flow. LL. Differentiate between forced vortex and free vortex flow. Kinematics of Flow and Ideal Flow 255 12, Derive an expression for the depth of paraboloid formed by the surface of a liguid contained in a cylindrical tank which ig rotated at a constant angular velocity «@ about its vertical axis. 13, Derive an expression for the difference of pressure between two points ina free vortex flow. Does the Gifference of pressure satisty Bernoulli's equation ” Can Bernoulli's equation be applied to a forced vortex flow ? 14, Derive, from first principles, the condition for irrotatonal flow. Prove that, for potential flow, both the stream function and velocity potential function satisfy the Laplace equation, 15, Define velocity potential function and stream function 16. Under what conditions can one treat real fluid flow as irotational (as an approximation) 17. Define the following (0 Steady flow, (i) Non-uniform flow, (iid) Laminar flow, and (iv) Two-dimensional flow 18, (@) Distinguish between rotational flow and irrotational flow. Give one example of each (®) Cite two examples of unsteady, non-uniform flow. How can the unsteady flow be transformed to steady flow ? (LNT. University, § 2002) 19. Explain uniform flow with source and sink. Obtain expressions for stream and velocity potential funetions, 20. A point source is a point where an incompressible fluid is imagined to be created and sent out evenly in all dictions. Determine its velocity potential and stream function, 21. (@ Explain doublet and define the strength of the doublet Gi) Distinguish between a source and a sink. 22, Sketch the flow pater of an ideal fluid flow past a eylinder with cireulation. 23. Show that in case of forced vortex flow, the rse of liquid level at the ends is equal to the fall of liquid level atthe axis of rotation. 24, Differentiate between (@ Stream function and velocity potential function (ii) Stream line and streak tine and id) Rotational and irrotational flows, (B) NUMERICAL PROBLEMS 1. The diameters of a pipe atthe sections I and 2 are 15 em and 20 em respectively, Find the discharge trough the pipe if Velocity of water at section I is 4 mvs, Determine also the velocity at section 2. [Ans. 0.07068 m*/s, 2.25 m/s} 2. 40 cm diameter pipe, conveying wate, branches into two pipes of diameters 30 em and 20 em respectively. I the average velocity inthe 40 cm diameter pipe is 3 nus. Find the discharge inthis pipe ‘Also determine the velocity in 20 em pipe ifthe average velocity in 30 em dlameter pipe is 2 m/s. (Ans. 0.3769 m's, 7.5 ms) 3. A.30 cm diameter pipe caries ol of sp. gr 0.8 a a velocity of 2 m/s, AL another section the diameter is 20 ci, Find the velocity at this section and also mass rate of low of oi, (Ans. 4.5 mf, 113 kgA] 4. The velocity vector ina fluid flow is given by V= 2x} — Sy) +41, Find the velocity and acceleration ofa fluid partite at (1, 2, 3) at time, = 1. [Ans 10:95 units, 16.12 units} 5. The following cases represent the 1wo velocity components determine the third component of velocity such that they satisfy the continuity equation [256 Fluid Mechanics ] Te 12. 1. 4 15. 16. 7 Ou = day (i) w= 4x + Bay, wae day — 292. try 4392 4705.2] [Ans.() w=—8x2— 227 +059 Ci) Calculate the unknown velocity components so that they sais the following equations @ ua2e,va2e, wa? Gun2e 42a, weed diet 2;e,0—2 [Ams (w= Lee Z] Prove that itis a case of possible steady incompressible Aid flow. Calculate the velocity and acceleration at the point [1 2, 3} [Ans. 36.7 units, 874.50 units} Find the convective acceleration at the middle of a pipe which converges uniformly from 0.6 m diameter t0 0.3 m diameter over 3 m length. The rate of flow is 40 lis. I the rate of flow changes uniformly from 40 livs to 80 livs in 40 seconds, find the total acceleration at the middle of the pipe at 20th second. [Ans 0499 mus; 11874 mi ‘The velocity potential function, 6, is given by § = .x°— y7. Find the velocity components in x and y direction. Also show that 6 represents a possible case of fluid flow. (Ans. u = 2x and v=~2)) . For the velocity potential function, @= 1° — y* find the velocity components atthe point (4, 5) [Ans. v= 8, =~ 10 units] A stream function is given by = y = 2x ~ Sy. Calculate the velocity components and also magnitude and ireotion of the resultant velocity at any point. [Ans. w= 5, v= 2, Resultant = 5.384 and 8 = 21° 48) If for a two-dimensional potential flow, the velocity potential is given by : = 4x(3y — 4), determine the velocity at the point 2, 3). Determine also the value of stream funetion y at the point (2, 3). [ms 40 ons ‘The stream function for a two-dimensional ow is given by y = Bay, calculate the velocity atthe point (4,5). Find the velocity potential function @. [Ans =—32 units, v= 40 units, = 4y? 4x7] ‘Sketch the stream lines represented by y =.xy. Also find out the velocity and is direction at point (2,3). [Ans. 3.60 units and 6 = 56° 18.6’ or 123° 42’) For the velocity components given as :u = ay sin xy, v= ax sin ay. Obtain an expression for the velocity potential function Ans. ¢ [A fhid flow is given by : V= 102% — By). Find the shear strain rte and state whether the flow is rotational or irvotational. (Ans. ~ 8x, rotational] cos xy] The velocity components in a two-dimensional flow are w=sey-8 Pandye—ary' + 8 x v8 ay s8 Show that these velocity components represent @ possible ease of an irrottional flow. Bu, av Ans. ‘An open circular cylinder of 20 em diameter and 100 em long contains water upto a height of 80 em. It is rotated about its vertical axis. Find the speed of rotation when : ( no water spills, (i axial depth is zero. (Ans. ()) 267.51 rpm. (i) 422.98 rpm] A cylindrical vessel 15 em in diameter and 40 em long is completely filled with water. The vessel is open atthe top. Find the quantity of water left in the vessel, when it is rotated about its vertical axis with a speed of 300 rpm. [Ans. 4566.3 em?) Kinematics of Flow and Ideal Flow _ 257 19. An open circular cylinder of 20 em diameter and 120 cm long contains water upto a height of 80 cu. Tt js rotated about its vertical ais at 400 rpm. Find the difference in total pressure force (atthe bottom, of the eylinder, and (i) atthe sides ofthe eylinder due to rotation. (Ans. (i) 14.52 N, (i) 2465.45 N} 20, A closed cylindrical vessel of diameter 15 cm and length 100 cm contains water upto a height of 80 em, The vessel is rotated ata speed of $00 rpm, about its vertical axis. Find the height of paraboloid formes. TAs. 56.06 em] 21. For the data given in question 20, find the speed of rotation of the vessel, when axial depth is zero. (Ans. 891.7 pm] 22. Ifthe cylindrical vessel of question 20, is rotated at 980 r-pam. about its vertical axis, find the area uncovered atthe base of the tank Ans. 20.4 em] 23. A closed cylindrical vese! of diameter 20 om and height 100 cm contains water upto a height of 70 em. The air above the water surace is ata pressure of 78.48 KN. The vesel is rotated a a speed of 300 rp. about its vertical axis, Find the pressure head atthe bottom of the vessel ; (a) atthe centre, and (D) at the edge [Ams (a) 8.4485 m () 89515 m] 24. A closed cylinder of diameter 30 em and height 20 em is completely filled with water. Caleulate the total pressure fore exerted by water on the top and bottom of the eylindr, if iti rotated about its vertical axis a 300 span, (Ans. Fp= 392.4 N, Fy= 531 NI 25. In free cylindrical vortex flow of water, ata point ata radius of 150 mm the velocity and pressure are ‘S mis and 14.715 Niem*. Find the pressure at a radius of 300 mm. [Ans. 15.65 Neem") 26. Do the following velocity components represent physically possible flows ? wart t5,vayt2,wednz [Ans. No] 27. State ifthe flow represented by u= 3x + 4y and v= 2x ~ 3y is rotational or itrotational, [Ans Rotational] 28. A vessel, eylindrieal in shape and closed at the top and bottom, contains water upto a height of 700 ram, The diameter of the vessel is 200 mm and length of vessel is 1.1 m, Find the speed of rolation ofthe vessel if the axial depth of water is zero 29 Define rotational and irrtational flow. The steam function and velocity potential fora flow are given ty y=20y, b= 27-y Show thatthe conditions of continuity and irrotational flow are satisfied. 30, For the steady incompressible lw, are the following Values of and v possible ? () w= dy ty ve Oxy + Brand Gi u=20 +y?,v=—day. [Ams (No, (ii) Yes} 31. Define two-dimensional stream function and velocity potential. Show tha following stream funtion y =6r-4y +749 represents an irrotational flow. Find its velocity potential. [Ans. §= 4r + 6y-3.5° +3.5y°+ C] 32. Check if 6 = 1° — y* + y represents the velocity potential for 2-limensional irrotational flow. If it does, then determine the stream function . [Ams. Yes, y =~ 2xy +x] 33. If stream function for steady flow is given by y= ()* ~ x"), determine whether the flow is rotational or inrotational, Then determine the velocity potential @, Ans. Irrotational, @=~ 2xy + C) 34. A pipe (1) 450 mm in diameter branches into two pipes (2) and (3) of diameters 300 mm and 200 mm respectively as shown in Fig, 5.57. If the average velocity in 450 mm diameter pipe is 3 m/s, find (@ discharge through 450 mm dia. pipe and (ii) velocity in 200 mm diameter pipe if the average velocity in 300 mm pipe is 2.5 m/s. (NT.U., Hyderabad, § 2002) (Hint. Given 4, =450mm=0.45 m, d;=300mm=0.3m 4, =200 mm =0.2.m, V;=3 mls, Vy=2.5 mis [258 Fluid Mechanics @ w But Also =F (045%) x3 0.477 mi, Fig.5.57 yo 925001760 1s = Q ~ Q)= 0.477 - 0.176 = 0.301 =F 02x", Qs 0.301 ag 0.0314 F023)

You might also like